Liabilities, Equity, and Corporate Groups - University of Alberta

66
CHAPTER NINE 9 Chapter Introduction: Accounting For the Right-Hand Side of the Balance Sheet Liabilities, Equity, and Corporate Groups 9.1 NEL H enry Woo, CEO of Thornton Cunliffe Inc. (TCI), and Sebastien Cruz, CEO of West Pass Industries Ltd. (WPIL) announced today that TCI has agreed to acquire the outstanding voting shares of WPIL. The combined strengths will position TCI to be first in Canada in serving its valued customers and will create a dynamic vehicle for investors. Both companies’ boards of directors voted unanimously for this combination of two strong and highly regarded businesses. Henry Woo will remain CEO of TCI, and Sebastien Cruz will become chairman of the board. Donna Gladred will become CEO of WPIL, which will continue to operate under its own name. To achieve the operating efficiencies envisaged, some WPIL units will be merged with TCI units, and the TCI Logistics Group will be transferred to WPIL. Gaining such efficiencies will require the elimination of some jobs at both companies, but normal attrition and turnover should provide for this so that no layoffs are expected. Under the terms of the acquisition agreement, owners of WPIL shares will receive an immediate cash dividend of $1.20 per share and will exchange their shares for newly issued voting shares of TCI on a 2.4 to 1 ratio. At current share prices on the Toronto Stock Exchange, this values WPIL at C$319 million. The fair values of WPIL’s net assets are estimated to be C$233 million, therefore goodwill on acquisition is valued at C$86 million. This represents the value of the WPIL name and expertise, as well as the synergies that will be achieved in the combined businesses. TCI will guarantee certain of WPIL’s debts and will under- write the revision of WPIL’s pension plans to make them consis- tent with TCI’s. Managers and directors of both companies will be eligible for a new stock option plan, under which benefits are contingent on increases in TCI’s share price looking forward three to five years. Contact Zoe Duran at TCI for a press kit with full details. News Release Thornton Cunliffe Inc. For release to all media at 4:00 p.m. Eastern Standard Time, November 22

Transcript of Liabilities, Equity, and Corporate Groups - University of Alberta

CHAPTER NINE 9Chapter Introduction: Accounting For the Right-Hand Side ofthe Balance Sheet

Liabilities,Equity, andCorporate Groups

9.1

NEL

Henry Woo, CEO of Thornton Cunliffe Inc. (TCI), andSebastien Cruz, CEO of West Pass Industries Ltd.(WPIL) announced today that TCI has agreed to

acquire the outstanding voting shares of WPIL. The combinedstrengths will position TCI to be first in Canada in serving itsvalued customers and will create a dynamic vehicle for investors.

Both companies’ boards of directors voted unanimously forthis combination of two strong and highly regarded businesses.Henry Woo will remain CEO of TCI, and Sebastien Cruz willbecome chairman of the board. Donna Gladred will become CEOof WPIL, which will continue to operate under its own name. Toachieve the operating efficiencies envisaged, some WPIL units willbe merged with TCI units, and the TCI Logistics Group will betransferred to WPIL. Gaining such efficiencies will require theelimination of some jobs at both companies, but normal attritionand turnover should provide for this so that no layoffs areexpected.

Under the terms of the acquisition agreement, owners ofWPIL shares will receive an immediate cash dividend of $1.20 pershare and will exchange their shares for newly issued voting sharesof TCI on a 2.4 to 1 ratio. At current share prices on the TorontoStock Exchange, this values WPIL at C$319 million. The fairvalues of WPIL’s net assets are estimated to be C$233 million,therefore goodwill on acquisition is valued at C$86 million. Thisrepresents the value of the WPIL name and expertise, as well asthe synergies that will be achieved in the combined businesses.

TCI will guarantee certain of WPIL’s debts and will under-write the revision of WPIL’s pension plans to make them consis-tent with TCI’s. Managers and directors of both companies will beeligible for a new stock option plan, under which benefits arecontingent on increases in TCI’s share price looking forwardthree to five years.

Contact Zoe Duran at TCI for a press kit with full details.

News ReleaseThornton Cunliffe Inc.

For release to all media at 4:00 p.m. Eastern Standard Time, November 22

FinancialAcct-C09-v3 3/28/06 2:56 PM Page 571

Chapter 8 was about the “left-

hand side” of the balance sheet

(assets). This chapter is about

the “right-hand side” (liabilities

and equity), and, because

Chapter 10 covers analysis of the financial statements, is the

final step in meeting the book’s overall objective that you be

able to pick up a company’s set of financial statements and

understand their contents. As has happened sometimes

already, meeting this objective requires brief attention to some

issues that are very complex and that are dealt with more thor-

oughly in advanced accounting courses. Most sections in this

chapter are not detailed, for two reasons: we have encountered

many of the issues already when considering asset and income

accounting, and the accounting practices for many liabilities

and equity items are very complex, beyond what this introduc-

tory book can sensibly cover. The intent is that you will learn

enough to make sense out of statements’ content without

drowning in details. Perhaps that will spark your interest to

learn more on your own or in other courses.

In this chapter you will learn:

• How companies’ legal and financial structures are

reported in the balance sheet. The right-hand side of the

balance sheet is where much of the complexity of modern

financial arrangements has to be worked out. This adds to

Section 2.9’s coverage of various forms of business: propri-

etorships, partnerships, and especially corporations;

• The basics of accounting and disclosure for various kinds

of legal debts and estimated accruals for such phenomena

as income taxes, pensions, and warranties;

• The principles behind accounting and disclosure for

equity, supplementing what you already know about share

capital and retained earnings;

• The nature of some controversies about liabilities and

equity that may not be reported very well in financial

accounting, such as financial instruments and stock

options of the sort to be awarded by Thornton Cunliffe in

the news release above;

• The basics of accounting for corporate groups, such as the

above combination of Thornton Cunliffe and West Pass

Industries. Accounting for corporate groups has a signifi-

cant impact on income measurement and on the asset side

of the balance sheet too; however, the topic is covered in

this chapter because it raises important issues that are

specific to the right-hand side. It is an example of an

accounting method that is very complex in practice but

has principles that are understandable without the practi-

cal complexities. It is also part of the goal of understand-

ing a corporate balance sheet, because, as we saw in

Chapter 2, most companies are actually corporate groups.

As for the asset accounting in Chapter 8, many of this chap-

ter’s topics have been introduced in earlier chapters, so this

chapter builds on earlier material and ties up a few loose ends.

Learning Objectives

572 PART THREE DOING FINANCIAL ACCOUNTING

Current LiabilitiesIn much of liability accounting, the principal difference between current and noncur-rent liabilities is just their timing. A loan due to be repaid in five months is shown as acurrent liability, and one due in five years is a noncurrent liability. Their due dates maybe the primary feature that distinguishes them. Similarly, an accrual for an expense thatis expected to be paid in five months is a current liability, and one that is expected to bepaid in five years is a noncurrent liability. Both are accruals used for income measure-ment; they may also differ primarily in timing. Because noncurrent liabilities tend to beharder to estimate as the future is farther away, there may be more practical complexi-ties for noncurrent liabilities than for current ones. Therefore, this section will examinesome accounting principles for liabilities, current and noncurrent, and will point outsome particular current liability issues. Noncurrent liability issues are picked up again inSection 9.3.

9.2

NEL

Many accountingprinciples apply toboth current andnoncurrent liabilities.

This chapter focuses onunderstanding the right-hand side of a typicalcorporate balance sheet.

FinancialAcct-C09-v3 3/28/06 2:56 PM Page 572

Even in such a traditional idea as separating current and noncurrent liabilities therecan be controversy. The distinction should be based on the expected pattern ofpayment, not on intention. For example, if a company has a debt that is due in sixmonths, it is current, even if management hopes to delay its payment for a couple ofyears. This simple idea has caused a fuss lately, especially in the case of “callable debt,”which is debt that is due at the option of the creditor. An example is that most bankloans in Canada are legally “demand loans.” For such loans, the bank can ask for itsmoney back with just a few days’ notice. Many companies have bank loans that theyrepay on a planned schedule, or that they constantly repay and reborrow as the needarises. Looking at the pattern of payment, you could say that such bank debt is reallynoncurrent because all or most of it is likely to be paid well into the future. But legally,no matter what management may intend, the creditor controls the repayment scheduleand can ask for the money on short notice. There is recent pressure to call all suchcallable debt current, no matter what the intended repayment schedule. Doing thiscould result in severe apparent damage to companies’ balance sheets if bank debt thathad been classified as noncurrent is moved to the current liability category.

Getting the current liabilities right is important for several reasons. The totalcurrent liabilities are part of the calculation of working capital and the working capitalratio, very important in assessing an enterprise’s financial strength. Many of the currentliability accounts are accruals of expenses, so getting income measured properlyrequires getting the accruals right. Conservatism supports recording all legitimate liabili-ties, especially any due in the short run or any whose recording would increase expenses(or reduce revenue). Doing this avoids overstating income. Conservatism also supportstransferring any short-term part of noncurrent debts into the current category, as in theexample above of callable debt, so that the short-term demands on cash to meet thosedebts are recognized.

As is true of assets, liabilities are significant both for their effect on balance sheetvaluation and their connection to income measurement. Their principal effect onincome measurement is through their association with expenses. Expenses arise fromconsuming the economic value of assets, such as inventory or fixed assets, but also fromincurring liabilities. Such liability incurrence arises from expense recognition prior to thecash flow, such as accounts payable, income tax payable, pension liability, and warrantyliability, topics mentioned in earlier chapters and examined in this chapter. Liabilitiesare sometimes associated with revenues too, such as via the deferred revenue liability for revenue collected before it is earned, but their main importance to income measurement is through expenses.

CHAPTER 9 LIABILITIES, EQUITY, AND CORPORATE GROUPS 573

NEL

Thecurrent–noncurrentdistinction should gobeyond intention topay.

Getting currentliabilities right isimportant inmeasuring incomeand working capital.

When auditors are examining the accounts, they payparticular attention to ensuring that no currentliabilities have been left out of the balance sheet.

They ask banks to provide written confirmation of loans, theyreview payments in the next period to see if any are for liabil-ities and corresponding expenses that should have beenrecorded in the current period, and they check accruals forunpaid wages, income taxes, interest, and other expenses.Sometimes extensive searches are made for evidence ofunrecorded liabilities, especially when there have been otherindications of accounting errors or manipulations.

A scam in the other direction was uncovered when anauditor found that a client company, trying to get itsincome down to save tax, had recorded expenses in thecurrent year that were not actually incurred until the nextyear. The company had overstated current liabilities, surpris-ing the auditor, who had been looking for understatementsof them! The supplier of the supplies and services involvedhad assisted with the scam by obligingly dating invoicesweeks earlier than they should have been.

FYIFYI

Liabilities are part ofthe balance sheet, butalso affect income,mainly throughexpenses.

FinancialAcct-C09-v3 3/28/06 2:56 PM Page 573

This section summarizes some important things you should know about accountingpolicies for liabilities, as to their valuation on the balance sheet and their connection toincome measurement. Only some parts will be new to you, but you should find thesummary useful.

Legal DebtsBank loans, trade accounts payable, wages payable, Canada Pension Plan, and employ-ees’ income tax deducted from their pay due to governments, other employee deduc-tions and fringe benefits due, sales taxes collected and due to governments, bondeddebt, mortgages, asset purchase contracts, and other legal debts are recorded whenincurred and are reported at the amounts incurred (minus anything paid so far). Hereare just a few details:

• Historical cost accounting applies here too. The amounts shown are those that arosewhen the debt was incurred. This is normally the same amount as will actually bepaid, but sometimes it is not. (An example of where it is not, bonds issued at adiscount or premium, is included in Section 9.3.)

• There is no recognition of nonhistorical interpretations of the debt, even if theeconomic meaning of the debt would increase because of such recognition. Threethings therefore that are not recognized are:• Interest that will have to be paid but has not yet accrued (for example, if a debt

is due in two years, only the interest to date is added, not the interest for thenext two years).

• Inflation (even though being in debt during a period of inflation is a good ideabecause you pay back with dollars worth less than those you borrowed).

• Market value changes in public debt (for example, if interest rates have risen somuch that a bond issued for $1,000 but now paying an unattractive interest rateis now selling on the bond market for only $780, the debt liability is not revalued on the balance sheet to reflect the lower market value).

• Unless there is evidence to the contrary, the company is assumed to be a goingconcern and, therefore, debts are shown at the amounts that would normally be paid,and are expected to be paid, not at some other liquidation value that might benegotiated with creditors if the company got into serious financial trouble.

• For important debts, some of the legal details are disclosed (usually in the notes tothe financial statements). The main details here are the interest rate on the debt(especially for noncurrent debt), any assets or other securities given, repaymentterms, and any special conditions, such as being convertible to equity.

Current Portion of Noncurrent DebtsIn one way, current and noncurrent debts are just two parts of the same debt. In orderto determine current liabilities properly, and conservatively, GAAP requires that if thereis a noncurrent debt on which some payment is to be made within the next year, thatpayment be included in current liabilities. So a single debt is split into two parts: currentand noncurrent. This does not affect the legal debt in the slightest: it is just done foraccounting purposes.

There’s a twist here you should watch for, consistent with the principle noted aboveof not recognizing future interest. In accordance with the above points, it’s only theprincipal portion payable in the next year that’s called current. Suppose, for example

• Jocelyn owes $71,000 on her mortgage.• During the next year, she must make 12 monthly payments of $1,000, including

interest.

574 PART THREE DOING FINANCIAL ACCOUNTING

NEL

Debts are shown atthe historical valuethat arose when thedebt was incurred.

Debts do not includefuture interest,inflation, or marketvalue changes.

Liabilities’ valuationassumes that theenterprise is a goingconcern and will pay.

Notes usually containdetails of importantdebts, especiallynoncurrent ones.

The current part ofnoncurrent debt isonly the principalportion of next year’spayments.

FinancialAcct-C09-v3 3/28/06 2:56 PM Page 574

• If the interest will amount to $6,400 over the next year, then the rest of the $12,000to be paid will be $5,600 on the principal.

• Therefore, her balance sheet will show two components of the $71,000 principalowing:• a current liability of $5,600, the amount of the principal to be repaid next year,

and• a noncurrent liability of the remaining $65,400 ($71,000 – $5,600).

• The current liability is not $12,000, the total of the payments to be made in the nextyear, because that includes the coming year’s interest, which is ignored in thebalance sheet because it has not yet accrued.

Short-Term Accruals and Other Current LiabilitiesAccrued interest, estimated after-sale service costs, estimated income tax payable, andother such estimated but not yet legally payable short-term liabilities are accounted forby debiting an expense account and crediting a current liability. Although they are notyet actual debts, they are reported in the same way as the legal debts. These accruals arenot controversial for current liabilities, but can be controversial if they are noncurrent,as Section 9.3 will show. Such accruals are a product of the matching process behindincome measurement, and they are usually done very carefully, because if they are not,an imprecise “cut off” of the expenses involved would make both the current year’s andnext year’s income wrong.

Current liabilities also include other credit balance accounts. • One, already mentioned above, and explained in Chapter 6, is deferred revenue or

customer deposits, which represent revenue collected before it has been earned.This is not necessarily a legal debt, but it is viewed as an economic one, in that theenterprise has not yet earned the money. In a business sense, it is a debt, because itwould be a poor business practice to collect money in advance from customers andrefuse to either do the agreed revenue-earning work or return the money.

• Another credit balance account, which was described in Section 6.10, is an accruedliability, the negative version of prepaid expense: an account that may usually be aprepaid expense but sometimes ends up as a credit instead, because the expense isincurred but for some reason is not paid in advance.

• Similar to this is a third kind of credit balance account, an asset that has gone nega-tive due to an event that is not typical of the asset. Two common examples are abank overdraft (overspent bank balance) and a credit balance in accounts receiv-able resulting from a customer overpaying the account. This last is like a customerdeposit but usually results from inadvertent overpayment or a billing error. (Theenterprise may overpay an account payable for the same reasons: if so, the debitbalance in accounts payable should be transferred to accounts receivable.)Reclassification of accounts between current liabilities and current assets is important only if the amounts involved are material to the total of either category.

CHAPTER 9 LIABILITIES, EQUITY, AND CORPORATE GROUPS 575

NEL

Short-term accrualsare for incomemeasurement andare usually notcontroversial.

Current liabilities alsoinclude variousmiscellaneous creditbalance accounts.

Here are two questions you should be able to answer basedon what you have just read. If you can’t answer them, itwould be best to reread the material.

1. Current liabilities arise for several reasons. What are they?2. Why is it important to get the current liabilities measured

properly?

how’s your understanding?

FinancialAcct-C09-v3 3/28/06 2:56 PM Page 575

Noncurrent Liabilities

Long-Term DebtsDebts that are due more than a year into the future are included in noncurrent liabili-ties, minus any principal portion due within the next year that is included in currentliabilities.

Most noncurrent liabilities are supported by specific agreements about repaymentterms between the enterprise and its lenders. These usually involve some security toprotect the lender. There are several common kinds of security, which can exist in various combinations.

• One is a mortgage held by the lender on the enterprise’s property or equipment sothat the lender can claim title to those assets if the enterprise does not make theagreed payments on time.

• A second kind of security is a debenture, which is a more general kind of right by thelender to take some degree of control over the enterprise if necessary.

• A third kind is an indenture, which is a set of specifications that the enterprise mustmeet otherwise the lender can demand payment or take other action. Such specifi-cations may be that the enterprise maintain a particular level of working capital, or aparticular working capital ratio, or meet other conditions defined on the financialstatements. (Such indentures may tempt management to choose accounting policiesdesigned to help the financial statements meet the agreed specifications.)

• A fourth kind of security, often used by banks with smaller company borrowers, is toask the owners of the company to provide personal guarantees in case the enterprise’sassets are not sufficient to repay the debt if trouble comes.

GAAP require some disclosure of important security on long-term debts, plus repaymentterms and some other details, so the financial statement notes about long-term debtscan be extensive. Some long-term debts, such as loans from shareholders, may be unse-cured and have an unspecified due date, which will also be disclosed if informative.

A common kind of noncurrent debt is an agreement to pay for an asset over aperiod of time.

• For land or buildings, such an agreement is usually a mortgage. • Equipment and vehicles may also be acquired with such agreements, often by a

conditional sale contract, under which the title does not pass to the enterprise until ithas met all the payments. These are usually not large in comparison to othernoncurrent liabilities, so there may not be much disclosure about them.

• Another way of acquiring economic assets, explained in Section 8.14, is via capitalleases. The liability for such leases, after being recorded by the process described inSection 8.14, is included in noncurrent (and current) liabilities just as if it was aregular debt, because the accounting principle is that they are economic debts.

Discounts or Premiums on Noncurrent DebtsSometimes noncurrent debt is issued at a discount or a premium. This is easiest toexplain with bonded debt. A bond is an instrument like a share that usually can be tradedamong investors, but instead of carrying ownership rights, carries a portion of a mort-gage, indenture agreement, or other security and has a limited term before it must berepaid and has the right to interest in the meantime.

• Suppose the enterprise decides to borrow using a bond issue composed of $1,000bonds carrying 7% interest.

9.3

576 PART THREE DOING FINANCIAL ACCOUNTING

NEL

Noncurrent debt isusually secured insome way, and thesecurity and otherdetails are disclosed.

Conditional salescontracts and capitalleases are commonkinds of noncurrentliabilities.

Discounts andpremiums result fromdifferences betweenbonds’ interest ratesand market rates.

FinancialAcct-C09-v3 3/28/06 2:56 PM Page 576

• When the bond issue is all ready, interest rates in the market for such bonds mayhave risen a bit, say to 8%. Lenders would not want a 7% bond.

• So the enterprise sells the bonds at a discount, a lower price such that the amountthe lender pays will earn 8%.

• The lender gives the company less than $1,000 for each bond, and that loweramount is such that the $70 interest (7% of $1,000) represents the 8% the lenderwants.

• If the interest rates have fallen, say to 6%, the lender will be willing to pay more than$1,000 for each bond, such that the $70 interest represents the 6% return the lenderwants. So the enterprise gets a premium for the bonds, more than $1,000 each.

(The present value calculations behind bond prices are included in Section 10.7.)

Example• A corporation issued 10,000 $1,000 bonds, thus having a total legal debt of

$10,000,000.• Suppose the bonds sold for a total of either $8,760,000 (a discount) or $11,180,000

(a premium).• The selling price can be said to be the appropriate price for that bond at prevailing

market interest rates, so in the first case, the bond pays interest at a rate belowmarket rates, and in the second case, pays at a rate above market rates.

• At the issue date of the bonds, the proceeds and discount or premium are recordedthus:

Discount Premium

Dr Cash (proceeds) 8,760,000 Dr Cash (proceeds) 11,180,000Cr Bonded debt 10,000,000 Cr Bonded debt 10,000,000

Dr Bond discount 1,240,000 Cr Bond premium 1,180,000

The bonded debt account is a liability. But so is the discount or premium. The premiumor discount account works as a contra account, to change the balance sheet valuation ofthe liability without changing the legal debt account. (The premium is a credit balanceaccount, so it is not opposite in sign as contra accounts like the allowance for doubtfulaccounts and accumulated amortization are.) The legal debt is what has to be repaid;the discount or premium is just an adjustment to get the proceeds to what will bring thebond market the return it requires. So on the day of issue, the enterprise’s balance sheetwould show a liability called bonded debt, at the amount of:

• $8,760,000 (in the case of the discount: $10,000,000 – $1,240,000), or • $11,180,000 (in the case of the premium: $10,000,000 + $1,180,000).

Thus the reported liability meets the historical cost criterion: it is what was received forthe bonds.

But the amount of the proceeds is not what will eventually be repaid to the lenders.That is $10,000,000 in both cases.

• So the discount or premium is amortized over the period until the bonds are due. Ittherefore shrinks away until on the due date it is zero and the $10,000,000 iscorrectly shown as the debt on that date.

• The period’s amortization amount is included with interest expense reported on theincome statement.

• The discount is a debit, so amortizing it adds to the interest expense, making thereported expense higher than the $70 cash interest paid per bond. This makessense, as the reason for the discount is that the bond market demanded a rate

CHAPTER 9 LIABILITIES, EQUITY, AND CORPORATE GROUPS 577

NEL

The discount orpremium is includedwith the legal debt onthe balance sheet.

Amortizing a bonddiscount or premiummakes the interestexpense approximatethe market rate.

FinancialAcct-C09-v3 3/28/06 2:56 PM Page 577

higher than 7%, and by selling the bonds at a discount, the enterprise provided thathigher rate. The real interest cost is higher than $70.

• In the case of a premium, the amortization reduces the reported interest expense,which again makes sense because the bond market was happy with a rate lower than7% and by selling the bonds at a premium, the enterprise provided that lower rate.

• Thus, the reported interest expense approximates the market rate demanded whenthe bonds were sold.

• If you already understand the concept of present value (Section 10.7), you will seethat what is happening is that the bonded debt, adjusted by the unamortizeddiscount or premium, is being shown on the balance sheet at the present value ofthe bond, calculated at the market interest rate in effect when the bond was issued.

• Methods for calculating amortization of a discount or premium are in moreadvanced accounting textbooks.

• Just like amortization on assets, amortization of discount or premium is a noncashexpense debit or credit that has to be adjusted for in the Operations section of thecash flow statement.

Long-Term AccrualsThese are in principle just longer-term versions of the short-term accruals.

• Like the current liability accruals, they are created by debiting an expense account. • But, since there will not be a payment for a long time, the credit is to a noncurrent

liability. • Many of these are approximate estimates, depending on many assumptions: they are

recorded in order to account for the future consequences of arrangements made tohelp earn income today, and so their main purpose is income measurement ratherthan balance sheet valuation.

• Referring to the relevance-reliability tradeoff, imprecise estimates of future paymentsare thought relevant to users of the financial statements even if they are not entirelyreliable.

For long-term accruals (often called provisions), there is no debt payable now and it’soften anyone’s guess as to exactly when a debt will arise and precisely how much it willbe. If feasible, such accruals are based on the present value of future estimated cash flowsbecause of the principle, mentioned in Section 9.2, of not recognizing future interest inthe balance sheet liability figure. Using present value is more feasible for accruals basedon contractual arrangements, such as pension liabilities, than those based on estimatesof more discretionary cash flows, such as product warranties. (The present value repre-sents the principal value of the liability, not including future interest, like a mortgagethe total payments of which exceed the present principal because the future paymentswill include interest. See Section 10.7.)

Warranty Liability• This is the estimated future cost of providing warranty service for products already

sold. The revenue is already recognized, so matching requires recognizing anypromises to fix problems that go with the revenue.

• In the period in which a product is sold, an expense is recognized to match to therevenue by the expense recognition entry:

DR Warranty expenseCR Warranty liability

• When a warranty payment is made, the liability is reduced by the payment entry:

DR Warranty liabilityCR Cash

578 PART THREE DOING FINANCIAL ACCOUNTING

NEL

Long-term accrualsare often imprecise,but are thoughtrelevant anyway.

Long-term accrualsare shown at thepresent value ofestimated futurepayments.

FinancialAcct-C09-v3 3/28/06 2:56 PM Page 578

Or, if a replacement product is provided:

DR Warranty liabilityCR Inventory

For the second payment entry to work out over time, the liability estimate would bebased on the cost of the products being provided as replacements, not on the sellingprice to the customers, the likely basis if cash payouts were to be made, as in the firstpayment entry.

• If, as is likely, some of the warranty cost will be paid within the next year, thatamount is included in current liabilities among the short-term accruals.

Pension Liability• This is the estimated future cost of providing pensions for work already done by

employees, minus cash paid to a pension trustee to be invested to fund the eventualpensions. (A pension trustee is a third party that manages the pension’s assets thatthe company paid out to the trustee. It is responsible for investing these assets andpayment of pension benefits to retired employees.)

• For example, if an employee has worked five years and is already entitled to somepart of a pension 30 years from now based on that work, the estimated present valueof that pension entitlement is recorded as a liability. Cash already paid to the trusteeis deducted from the liability.

• You can see the problems in trying to estimate such a liability when you think of allthe things that can change in those 30 years and that must be thought about inmaking the estimate: the employee might die first, be fired, or quit; interest rates(used in the present value calculation) will doubtless vary; the pension plan itselfmight change, even retroactively; laws governing such plans may change; and so on.

• When the employee earns a pension entitlement, that is presumed to be an expenseof the period in which it is earned and is recorded by the expense recognition entry:

DR Pension expenseCR Pension liability

• When a payment is made to a trustee or directly to a retired employee, the paymentis recorded by:

DR Pension liabilityCR Cash

CHAPTER 9 LIABILITIES, EQUITY, AND CORPORATE GROUPS 579

NEL

Here is an example of the difficulty of making a long-term accrual. Like the hippie music the radio stationsplay for the baby boomers, it’s an oldie but goodie.

In the 1960s, General Motors produced a little rear-enginedcar called the Corvair. It was popular and seemed trouble-free. There were Corvair clubs of devotees. It was all verysweet. GM did not have to have a very large warranty provi-sion for the Corvair. But then Ralph Nader’s famous bookUnsafe at Any Speed came out, criticizing the Corvair as wellas other cars. People returned to their car dealers in droves,

complaining about their Corvairs. It was no longer so sweet.Suddenly, GM had to increase its warranty provision,current, and noncurrent, because of the cost of fixing real orimagined problems. Its warranty expense estimates hadbeen fine under previous conditions, but were suddenlymade wrong by the unanticipated event of Nader’s book.It’s an example of the unavoidable fact that accrual account-ing estimates of the future, no matter how carefully made,can easily turn out later to have been wrong.

FYIFYI

Pension liability is agood example of acomplicated but stillimprecise estimate.

FinancialAcct-C09-v3 3/28/06 2:56 PM Page 579

This transfers the money to the trustee, and takes it out of the company’s accounts. So acompany might have a very small pension liability if it is keeping up with paymentsto the trustee. But it might have a contingent liability: having made promises to payemployees, the company normally would keep those promises even if the trustee didnot earn enough money investing the funds to make the promised pensionpayments years later. Contingent liabilities are disclosed in notes to the financialstatements—as you can see from this example, they also are based on estimates ofwhat might happen.

• Pension liabilities continually need adjustment as conditions and assumptionschange, and those adjustments are just included in wages or other expenses of theperiod of the adjustment, because they are a routine consequence of accrualaccounting, just as adjustments to asset amortization, allowance for doubtfulaccounts, and other estimates are routinely made.

• Because they are important to understanding a company’s promises to its employeesand its long-term financial situation, pension plans are usually described at length innotes to the financial statements, which normally include information about thetotal estimated obligation and the assets to meet those obligations that are held bythe trustee. CPR disclosed, in a note that took nearly two pages, that at the end of2004, the estimated obligation (present value) was $6.827 billion, much bigger thanany of its balance sheet liabilities. Because the obligation was more than covered, asCPR calculated it, the company showed an $838.3 million pension plan asset amongits noncurrent assets.1

• As the CPR example shows, if the company makes more payments than are neces-sary to meet the estimated needs of the pension plan, the pension liability canbecome an asset, a noncurrent prepaid expense. The pension liability can thereforechange to an asset, and back again, just like the prepaid and accrued expensesexplained in Section 6.10.

• There are three awkward complications with pension plans, much in the news lately:• The trustee’s pension assets and obligations are not included in the company’s

balance sheet, so that means the company may have very large off-balance-sheetassets and liabilities, as CPR does. Not only do these represent contingent liabili-ties (and contingent assets), but also their role in the company’s financial opera-tions may be very large and not apparent from the balance sheet, just in longtechnical notes.

• If the trustee has more funds than needed to meet the estimated obligations, sothat the plan is over-funded, whose money is the extra? The employees’ or thecompany’s? There have been major battles with unions and other employeegroups over this, because in the go-go 1990s, many plans earned more money ininterest and stock market gains than expected, and some companies took backthe extra to help with their general financing. The trustee’s assets and liabilities,and any surplus, are not included in the company’s balance sheet (thoughmaybe they should be). This is an example of the very controversial off-balance-sheet financing issue, about which there will be more in Section 9.5. In the earlyyears of the 2000s, many pension plans performed badly, even losing money, socompanies’ contingent liabilities looked likely to become real ones in order tomake up the shortfall.2

• Often the pension trustee invests the funds in the company’s own shares. This ishow CPR does it. This means that the employees’ ability to receive theirpensions depends on the company’s continued success. In the optimistic 1990s,this was thought to provide a good incentive to employees to want to make thecompany succeed, as well as being a source of equity financing for the company.But what happens if the company goes bust, so that the share values plunge or

580 PART THREE DOING FINANCIAL ACCOUNTING

NEL

FinancialAcct-C09-v3 3/28/06 2:56 PM Page 580

disappear? Then the employees get no pension. The company’s contingentliability becomes huge, but if it is bust, it will not be able to pay, and the employ-ees’ pension trustee is left with worthless share certificates to paper the wallswith. This is what happened in the Enron case in 2001: employees lost virtuallyall their pensions because the trustee had invested in Enron shares, which lostall their value when Enron collapsed.3

According to Financial Reporting in Canada 2005, 142 of its 200 surveyed companiesreported having some sort of pension plan in 2004. Of these, 38 disclosed a pensionliability, 5 a deferred asset, and 71 both, for a total of 114 disclosing some sort of balancesheet amount, up from 98 in 2001. Changes to improve disclosure requirements relatedto pension benefits are required by the CICA Handbook for years ending on or after June30, 2004.4 This may, at least in part, account for the increase in companies makingbalance sheet disclosures.

Post-Employment Benefits Other than Pensions• These are similar to pension benefits, including health care, dental care, life insur-

ance and other benefits. The differences from pension promises are often subtle,and the problems of estimating the cost of such promises are similarly vexing, somost of the points made above about pensions apply here too.

• The expense recognition and payment entries follow the same pattern as forpensions.

• Recent changes in Canadian accounting standards group these benefits withpensions, and similarly complex note disclosure is called for. CPR combined thesewith its pension disclosures, as the revised standards recommend.

• With Canada’s state support of medical costs, these obligations are usually not aslarge for Canadian companies as they are for companies in the United States, wherethe obligations can be billions of dollars. In its 2004 annual report, for example,IBM reported a noncurrent liability of $8.4 billion.5 Several news reports havediscussed the cost added to General Motors vehicles by the cost of post-retirementbenefits. In this regard GM reported an accrued liability of $28.1 billion at the endof 2004.6 CPR, by contrast, reported a liability of only $248 million.7

• According to Financial Reporting in Canada 2005, 96 of the 200 companies disclosedthat they had post-retirement benefit plans in 2004 in a separate note.

8

Future Income TaxesAccounting for future income taxes is one of the most challenging topics in financialaccounting. Part of the challenge results from the complexity of income tax law, andpart from great disagreements about how future income tax should be accounted for, ifat all. After many years of debate, the CICA Handbook was revised in 1997 to provide afundamentally different way of accounting for future income taxes than had been doneuntil then. (The new method was fundamentally different from the prior Canadianmethod, but more like the U.S. and IASC methods.) To emphasize the change inmethod, the name of the method and of the accounts involved was changed fromdeferred income tax to future income taxes. This section will outline the principles behind theaccounting method: you don’t have to be a tax expert to understand them!

• Here’s the problem. Because income tax law differs in many ways from financialaccounting principles, there are differences between the income tax liability you’dget from the accounting income and other accounting numbers, and the incometax payable to the government for the year. Suppose, for example, that the followingtemporary difference exists:

CHAPTER 9 LIABILITIES, EQUITY, AND CORPORATE GROUPS 581

NEL

The method ofaccounting for futureincome taxes hasrecently changed inCanada.

The income taxaccounting problemarises because oftemporary differencesbetween accountingand the tax law.

FinancialAcct-C09-v3 3/28/06 2:56 PM Page 581

• According to the best estimates made in the company’s income tax returns forthis year, the company owes $800,000 in income tax for the current year.

• The company has been deducting amortization for income tax purposes (calledcapital cost allowance) on an accelerated basis whereas the amortization in theaccounts is straight-line. As we saw in Section 8.12, the two amortization meth-ods eventually get to the same book value (salvage value, or zero), but there istemporarily a difference.

• At the end of last year, this difference multiplied by the income tax rate thecompany expected to pay was $1,100,000. This was recorded as future incometax liability at the end of last year.

• At the end of this year, the difference times the income tax rate was $1,250,000.The future income tax liability therefore has to be increased by $150,000 to thenew number.

• The income tax expense for the year then is $800,000 currently payable plus$150,000 to adjust the future income tax liability, totalling $950,000.

• The journal entries to record all this would be:

DR Current portion of income tax expense 800,000CR Income tax payable 800,000

DR Future portion of income tax expense 150,000CR Future income tax liability 150,000

• The income tax expense on the income statement is thus the result of adjusting twobalance sheet accounts: income tax payable and future income tax liability. The expenseon the income statement will be the sum, $950,000 in the example, but the twoamounts, current portion of income tax expense and future portion of income tax expense,should be disclosed in the notes if not on the income statement.

• On the cash flow statement, changes in income tax payable are part of the noncashworking capital adjustment in calculating cash from operations, and changes in thefuture income tax liability are part of the noncash expenses adjustment, like amortization and long-term accruals such as pensions.

• In practice, the way the future portion of the income tax expense is determined isby analyzing all the balance sheet accounts that could involve differences betweenthe accounting numbers and the income tax rules. The amortization versus capitalcost allowance difference illustrated above is the most common source of suchdifferences, but the income tax law is very complex, so there can be dozens of othersources of differences. Also, income tax is usually payable, but, as may havehappened to you, companies sometimes get tax refunds, so there may be an incometax receivable. The amortization versus capital cost allowance difference creates anoncurrent future income tax liability on most companies’ balance sheets, butbecause of the income tax complexities, it is possible, even likely, that the companywill have two, three, or even six income tax items on its balance sheet all at once(there may, for example, be tax payable in Canada, but a tax receivable from theUnited States through a branch or subsidiary):

Potential Income Tax Locations on the Balance Sheet

Current assets Current LiabilitiesIncome tax receivable Income tax payableFuture income tax (perhaps with Future income tax (perhaps with

prepaid expenses) accrued expenses)

Noncurrent assets Noncurrent liabilitiesFuture income tax (perhaps with Future income tax (usually shown

deferred charges) on its own)

Notes to the financial statements will usually disclose any of these that are material, especially if they are grouped with other assets or liabilities.

582 PART THREE DOING FINANCIAL ACCOUNTING

NEL

FinancialAcct-C09-v3 3/28/06 2:56 PM Page 582

• To illustrate, at the end of 2004, CPR had the following balance sheet accounts,which you can see in the balance sheet we studied earlier in Section 2.10:• A current asset for future income taxes of $70.2 million;• A current liability for income (and other taxes) payable of $16.2 million; and• A noncurrent liability for future income taxes of $1,386.1 million (over

$1 billion!).• Similarly, since all the income tax accounts can be assets (debits) or liabilities (cred-

its), the income tax expenses on the income statement can be any of these fourcombinations (the first one is the most common and was illustrated in the exampleabove):

Potential Combinations of Income Tax Expense Portions

Adjustment Required Resulting IncomeOn Balance Sheet Tax Expense

Current Payable Future or Receivable Liability or Asset Current Portion Future Portion Total Expense

CR CR DR DR DR + DRDR CR CR DR CR + DRCR DR DR CR DR + CRDR DR CR CR CR + CR

• If you’re not fed up with all this complexity, there is one more issue. Combiningboth current and future income tax involves spreading the expected tax burden outover many years, many accounting periods. That is called interperiod tax allocation,which you might remember from Section 3.5. But you might also remember fromSection 3.5 that there is intraperiod tax allocation, which involves allocating the totalincome tax expense to the continuing income, the discontinued operations,extraordinary items, retained earnings adjustments, etc. Well, all these intraperiodallocations must also be done for both current and future portions of the incometax expense. It keeps many accountants very busy!

Why go to all this trouble? The simplest solution would be just to use the income taxactually payable, $800,000 in the above example, as the expense for the year. Just DRIncome tax expense and CR Income tax payable. Then pay the tax. Easy. Many people inbusiness would not expect otherwise.

But accrual accounting doesn’t let us off the hook so easily. There is a big problem:the income tax expense would then depend on an amortization method other than theone in the accounts. The expense would not match the way other expenses areaccounted for in the financial statements (amortization in this example, but manyothers in the reality of complex modern income taxation).

The matching principle would be violated if income tax just included the presentyear’s tax payable, so accounting standards specify that in the case of temporary differ-ences like this, the income tax expense should match the rest of the accounting. Because the esti-mates of future income tax arise from balance sheet accounts (and for other reasons wewon’t go into, such as changes in income tax rates), they don’t necessarily exactly matchthe way revenues and other expenses were calculated. But they are close because, afterall, the revenue and expense recognition affects the balance sheet as the income state-ment articulates with it. Thus, tax allocation accounting using future income tax liabilities and assets is specified by accounting standards worldwide.

Well, one might say, horrified by the complexity outlined above, why not just use thetax amortization (capital cost allowance) in the accounts rather than the straight-line orother accounting methods, so that no temporary difference arises and there are then nofuture income tax implications? Simple, too, and it is appealing to businesspeople who

CHAPTER 9 LIABILITIES, EQUITY, AND CORPORATE GROUPS 583

NEL

The matchingprinciple requiresthat income taxaccounting followthe rest of theaccounting.

It wouldn’t work tojust use the incometax payable as theincome tax expense.

FinancialAcct-C09-v3 3/28/06 2:56 PM Page 583

don’t want to spend a lot of money on accounting—but too simple, unfortunately.There are many differences between accounting principles and income tax law, and sounless we are prepared to scrap GAAP and just account for everything exactly the sameas the tax law, we’re stuck with some differences.

It would be appealing to just use the tax law for accounting, but that is not thoughtappropriate, at least not in Canada, the United States, and many other countries. Thetax law is written for all sorts of economic, political, and other reasons besides measur-ing financial performance and position, so it would make a poor basis for accounting.For example, the government may say that machinery bought between July and Octoberof year 200X can be amortized for tax purposes faster than regular machinery, and thenin September announce that the deadline has been extended to December. Or thatmachinery located in an economically depressed part of the country should get a fastertax amortization than the same machinery located somewhere else. This is all quitereasonable for the government to do. But should companies keep changing theiraccounting every time the government makes such a decision, and should the samemachinery, with the same useful economic life, be amortized differently just because it islocated in a different province? You might argue, yes, sure, the machinery is a differenteconomic asset because it is in a different place or was bought in December rather thanJanuary. But after much study of the implications, involving many countries (with widelyvarying tax systems), over many years, accounting standard-setters have concluded that itwould be inappropriate for financial accounting’s income tax numbers and all therelated numbers such as amortization to ignore GAAP and be based on arbitrary tax lawvariations.

Income tax accounting uses whatever income tax rates apply in the variousprovinces or countries in which the company operates. By dividing the total income taxexpense, which is the combination of the current and future portions of the income taxexpense, by the continuing income before income tax, the enterprise’s effective incometax rate can be calculated. For example, if the current portion of the expense is $311,000and the future portion is negative $(73,000), the net total expense is $238,000. If theaccounting income is $680,000, then the effective income tax rate is 35% ($238,000 /$680,000). Companies are supposed to explain why their effective rate differs from theofficial “statutory” rate if the difference is significant. Some reasons include havingnontaxable revenues (dividend income, for example, is not taxed in Canada if receivedfrom another corporation that already paid income tax in Canada), having expensesthat are not tax deductible (there are quite a few of these, including portions of enter-tainment expenses, some political contributions, and some kinds of financingexpenses), or taking advantage of special industry provisions such as tax credits for oiland gas exploration.

Mini-Demonstration Problem: Income TaxAccountingAt the end of last year, Canadian Specific Railway (CSR) had the following income taxaccounts on its balance sheet:

Current assets Current liabilities

Income tax receivable $ 75,000 Income tax payable $ 614,000Future income tax $114,000 Future income tax $ 0

Noncurrent assets Noncurrent liabilities

Future income tax $329,000 Future income tax $8,743,000

584 PART THREE DOING FINANCIAL ACCOUNTING

NEL

Government incometax policies are likelyto have nothing to dowith accountingmeasurement.

Future income taxadjustments useeffective tax rates.

FinancialAcct-C09-v3 3/28/06 2:56 PM Page 584

At the end of this year, the company’s accounts showed:

Current assets Current liabilities

Income tax receivable $ 0 Income tax payable $ 237,000Future income tax $196,000 Future income tax $ 17,000

Noncurrent assets Noncurrent liabilities

Future income tax $612,000 Future income tax $9,229,000

During the year, CSR paid $5,218,000 in income tax to various governments andreceived a refund of $75,000. According to its income tax returns, the company’s esti-mated income tax due for this year was $4,841,000. What were the following figures onCSR’s income statement for this year: (a) current portion of income tax expense, (b) future portion of income tax expense, and (c) total income tax expense? Thecompany’s income before income tax on the income statement for this year was$14,226,000, so (d) what was its effective income tax rate? (e) What income tax adjust-ments would appear on this year’s cash flow statement?

(a) Current Portion of Income Tax ExpenseThis equals the estimated tax due for the year according to the tax returns: $4,841,000.The amounts paid or refunds collected are irrelevant to the income statement, which isprepared on an accrual basis, not a cash flow basis.

(b) Future Portion of Income Tax ExpenseThis equals the net changes in the future income tax accounts from last year to this:

Current asset: $196,000 – $114,000 $ (82,000)Noncurrent asset: $612,000 – $329,000 (283,000)Current liability: $17,000 – $0 17,000Noncurrent liability: $9,229,000 – $8,743,000 486,000

Net total $ 138,000

The future portion of the expense is the net total, $138,000. The changes in assets arenetted against the changes in liabilities: the liability increases represent increases in theexpense, while the asset increases represent decreases in the expense, just as is true forany other liabilities (such as accounts payable) or assets (such as inventories or prepaidexpenses).

(c) Total Income Tax ExpenseSum of current and future: $4,841,000 + $138,000 = $4,979,000.

(d) Effective Income Tax RateIncome tax expense $4,979,000 divided by income before income tax $14,226,000 =35%.

(e) Income Tax Adjustments on Cash Flow StatementAll the adjustments below would be in the Operations section of the cash flow statement, part of the conversion from net income to cash from operations:

• The net change in the receivable and payable accounts would be shown as part ofthe changes in noncash working capital: the asset change of negative $75,000 wouldbe added back to income because it helped cash, while the liability change of nega-tive $377,000 ($614,000 down to $237,000) would be subtracted because reducing ittook more cash. This would produce a net subtraction of $302,000 from income.

CHAPTER 9 LIABILITIES, EQUITY, AND CORPORATE GROUPS 585

NEL

FinancialAcct-C09-v3 3/28/06 2:56 PM Page 585

• The net future income tax expense (which came from changes in the four futureincome tax balance sheet accounts) would be added back to net income as anoncash expense along with amortization and other noncash expenses: $138,000. Itis possible that the net changes in the current and noncurrent future income taxaccounts would be shown separately, or the former included with the noncash working capital changes, but their sum would still add to $138,000.

• The result would be a net overall subtraction from income of $164,000 ($302,000 –$138,000). Cash flow was higher than the accrual expense, so the cash outflow forincome tax = the accrual expense of $4,979,000 + $164,000 = $5,143,000 outflow,according to the cash flow statement. (This would not be shown as a separate figureon the cash flow statement, but would be part of the overall conversion from accrualnet income to cash from operations.)

• Check: The net cash paid out for income tax was $5,218,000 paid – $75,000 received= $5,143,000.

EquityThis section reviews topics in accounting for equity of unincorporated businesses andcorporations. Some of these have been mentioned earlier in the book, and complexitiesare left for other books. Referring back to the equity descriptions in Section 2.9 may behelpful if your memory of the various forms of business and kinds of equity is hazy.Section 9.5 outlines ideas about complex financial structures and Sections 9.6 and 9.7are about corporate groups—all of these have some implications to equity accounting.

Unincorporated BusinessesUnincorporated businesses, sole proprietorships or partnerships, are not legal persons sepa-rate from their owners, as corporations are. But the accounting principle stated inChapters 2 and 3 of separating the company’s dealings with customers and suppliers(from which it earns income) from its dealings with owners is still followed. The result isthe following set of accounting practices for unincorporated businesses. Because ofdouble-entry accounting and the articulation of the income statement and balancesheet, such practices for assets, liabilities, revenues, and expenses affect equity too,directly or indirectly.

9.4

586 PART THREE DOING FINANCIAL ACCOUNTING

NEL

Here are two questions you should be able to answer basedon what you have just read. If you can’t answer them, itwould be best to reread the material.

1. List the main kinds of noncurrent liabilities you’d expect tosee in a typical balance sheet and describe how each kindis valued (how the number representing it is determined).

2. For this year, Kite Ltd.’s tax return estimates $32,118owing in income tax for the year, but if you applied itseffective tax rate to its income statement’s “income beforeincome tax,” you’d get $22,949. If you examined thedifferences between accounting and income tax law for itsassets and liabilities, you’d get an estimated future income

tax liability of $28,976 at the end of this year (it was$38,415 at the end of last year). What would be (a) thecurrent portion of income tax expense for this year, (b) the future portion, and (c) the total income taxexpense?

(a) Current portion = income tax return estimate = $32,118.

(b) Future portion = change in future tax liability = $28,976 –$38,415 = $(9,439).

(c) Total income tax expense $32,118 – $9,439 = $22,679 net.

how’s your understanding?

FinancialAcct-C09-v3 3/28/06 2:56 PM Page 586

• Except for specific variations, most of which are given below, the accounting forassets, liabilities, revenues, and expenses is the same as for corporations, and so isthe same as has been explained in earlier chapters. Unincorporated businesses stillhave accounts receivable, inventories, amortization, debts, short- and long-termaccruals, and other business trappings. Accounting for assets and most liabilities isvirtually unaffected by the form of business.

• Five main liability differences do exist, some of which affect the accounting forequity too. 1. Unincorporated businesses (and “private” corporations too) cannot issue debt

to the public, so you will not see bonded debt on the balance sheet. 2. Any sort of secured debt is really a debt of the owner, so it is a little arbitrary as

to whether something like a mortgage is included in the balance sheet—usuallyit is included if the mortgaged asset is included on the other side of the balancesheet, but there should be a note about the legal circumstances.

3. Because the business and the owner are not really separable, generally there isno such thing as an owner’s loan to the business corresponding to a share-holder’s loan. Any investment by the owner (or owners, in the case of a partner-ship) is just included in equity.

4. Similarly, the business cannot declare a dividend, so there is no such thing asdividends payable. The owner(s) may feel that some payment should be madeby the business, but until that payment is made, there has been no economictransaction. Any payments to the owner(s) are just deducted from equity whenmade.

5. Consistent with all this, and with income tax law, the business is not subject toincome tax. Income tax is a personal obligation of the owner(s). Thereforethere is no income tax expense on the income statement and no income taxpayable or future income tax liability on the balance sheet. If the business doespay income tax on behalf of the owner, that is just deducted from equity whenmade, because it is really just a withdrawal of cash by the owner(s), as is truewhenever the business happens to pay a personal bill for the owner.

• Two important expenses you’d see in a corporation are missing from the unincorpo-rated business’s income statement. There is no income tax expense and there is noexpense for the owner’s (or owners’) wages or salaries. Any payments for income taxor to owners are deducted from equity, not shown as business expenses. The lack ofan income tax expense in particular means that the income number is usually justcalled income, not net income. The lack of both of these expenses, coupled with theoccasionally arbitrary classification of assets like buildings and cars, and expenseslike travel and entertainment, as being part of the business or part of the owner’s(owners’) personal affairs, can make it difficult to compare the profitability of anunincorporated business to that of a corporation.

• The unincorporated business’s balance sheet has a simple equity section: usuallyshown as just one figure called capital and calculated as:

Beginning capital + new investment by owner(s) + income for the year (or minus loss) – withdrawals by owner(s) = ending capital.

There may be a statement of owner’s (or owners’) capital changes, something likethe statement of retained earnings except including all the equity changes listedabove, and for a partnership there is usually a statement or analysis of the partners’individual capital accounts, as was illustrated in Section 2.9.

• Notes to the financial statements, financial statement titles, and account names areusually used to explain the business’s relationship to the owner(s) and to coverissues such as those outlined above.

CHAPTER 9 LIABILITIES, EQUITY, AND CORPORATE GROUPS 587

NEL

Unincorporatedbusinesses havegenerally similar assetsand liabilities to thoseof corporations.

Any investment orwithdrawals by theowner(s) are includedin equity.

An unincorporatedbusiness’s incomestatement can be hardto compare to acorporation’s.

Unincorporatedbusinesses lump allthe equity itemstogether into a singlecapital figure.

FinancialAcct-C09-v3 3/28/06 2:56 PM Page 587

Incorporated Businesses For an incorporated company (a corporation or limited company), there are severalmainly legal requirements that influence the accounting:

• Dividends must ordinarily be paid out of accumulated income only and not out ofinvested capital. Therefore, shareholders’ equity is divided into invested capital(share capital) and retained earnings, and only the latter is available for dividends.

• Share capital shows the dollar amounts, for each class of shares, contributed byshareholders over the years to buy shares directly from the company, including iden-tifying any amounts paid in excess of par values (which may be called contributedsurplus or some other name indicating it’s an extra). These amounts are all historicalfigures. CPR had a contributed surplus in its equity section of $330.4 million in2004.

• Various legal details about classes of shares and their rights and restrictions aredisclosed separately, on the balance sheet and/or in a note.

• Retained earnings shows the accumulated income minus dividends declared sincethe company’s incorporation. Restrictions on dividends may be disclosed. Declareddividends are liabilities of the corporation until paid.

Some other items should be mentioned here:

1. Shareholders may lend the company money, for example, by advancing money thatthe shareholders want to have repaid rather than investing in permanent share capi-tal. In a small private corporation, the owner-manager may lend by not withdrawingall the salary that has been recorded as an expense, perhaps because the company isshort of money. Such a loan from shareholder(s) is shown under liabilities, notowners’ equity, because the shareholder is acting as a creditor rather than an owner(though the distinction may strike you as rather slight).

2. The company may (in some jurisdictions) buy a few of its own shares. Such sharesrepresent an interest in the assets and do not have voting rights. Therefore, suchshares are either cancelled, which reduces share capital, or else called treasury sharesand deducted from the rest of owners’ equity at the cost paid for them, at least untilthe shares are resold or cancelled (see Section 2.9).

3. Also in Section 2.9, an awkward equity account, accumulated foreign currency transla-tion adjustment, was explained. It’s a consequence of adjusting all the other balancesheet and income statement accounts for transactions in foreign currencies, assetspurchased with foreign currencies and liabilities owed in foreign currencies.Standards are changing so that in future it is likely the equity account will be elimi-nated by instead putting these adjustments into the income statement. For now,however, this account is a feature of the equity sections of most companies withoperations in other countries. CPR is no different as its balance sheet showedforeign currency translation adjustments of $77.0 million.

4. Two phenomena that cause a little fun in accounting for corporate equity are sharesplits and stock dividends. A split is when the corporation’s shares are divided intotwice or three times as many shares, so that their smaller value may be easier forinvestors to purchase. (If some venerable companies, such as IBM, had not repeat-edly split their shares over the years, each one would cost thousands or hundreds ofthousands of dollars on the market.) A stock dividend is when the board of directorsdecides to issue some new shares (a small percentage of those already outstanding)to the shareholders instead of paying a cash dividend. Note two points about these.(1) They are really just different versions of the same thing. A 2-for-1 share (stock)

split means that if you owned 100 shares, you now own 200. A 10% stock divi-dend means that if you owned 100 shares, you now own 110.

588 PART THREE DOING FINANCIAL ACCOUNTING

NEL

A corporation’sbalance sheet isaccompanied byconsiderable legaldetail about equity.

Shareholders of acorporation may becreditors too.

Treasury shares notcancelled are shownat cost as a reductionin shareholders’equity.

The foreign currencytranslation adjustmentis in equity for lack ofanother place to putit.

FinancialAcct-C09-v3 3/28/06 2:56 PM Page 588

(2) If the stock markets are efficient, splits and stock dividends should not affectshare values: the value of split shares should fall by 50%, so that the two newones equal the value of the one old one, and the value of shares after a 10%stock dividend should also fall so that 110% of the new shares are worth what100% of the old shares were. But the stock markets do not always react this way.Because a split makes the shares a little easier to own, the value of split sharesmay fall a little less than 50%, and the value of shares after a stock dividend maybe a little higher than just dividing the old value over the new number of shares.

The traditional accounting practices for splits and stock dividends depend on ajudgment that may be difficult to defend given modern stock market pricing. That judg-ment is that share splits provide no benefit to shareholders, but that stock dividends do.

(a) So if a corporation splits its shares, there is no accounting entry. The sharerecords and the notes to the financial statements are just changed to showthe new number of shares.

(b) On the other hand, a stock dividend is allocated a value: in the case of a10% stock dividend, an entry is made:

DR Retained earnings CR Share capital

This entry recognizes a deemed increase in the book value of share capital.Calculating the amount and distinguishing between a small split and a bigstock dividend are explained in more advanced accounting books.

Nonbusiness OrganizationsAs described in Section 5.5, nonbusiness organizations such as governments, clubs, andchurches typically have different accounting for equity because they don’t have owners,share capital, dividends, and other business trappings. When you are using the financialstatements of such an organization, pay particular attention to what kind of organizationit is and to the way that it is represented in the balance sheet equity section, becausethere is enormous variation that flows from tradition as well as legal particulars of eachorganization.

Generally, nonbusiness organizations have equity sections that are simpler thanthose of corporations. Often they show just a single figure calculated as total assets

CHAPTER 9 LIABILITIES, EQUITY, AND CORPORATE GROUPS 589

NEL

Most companies perform stock splits when there is afeeling that their stock has become too expensive.Commonly, many companies like to keep their

stock value under $100 per common share. One glaringexception to this practice is Berkshire Hathaway Inc.(www.berkshirehathaway.com), a holding companyheaded by Warren Buffett, which invests in and owns a vari-ety of companies. The company’s main activities are in theinsurance field but it has a variety of other subsidiariesincluding Dairy Queen, Benjamin Moore, and numerousothers. At the end of 2001, it had more than $500 million inequity investments including 8% of the common shares ofCoca-Cola and 9% of the common shares of the Gillette

Company, just to name a few.9 Berkshire Hathaway hasnever split its common shares and in mid-2002, a singleshare cost more than U.S.$70,000. For many people,purchasing one share might compose their entire invest-ment portfolio! (To find out Berkshire’s current price, go towww.nyse.com and search under the ticker symbolBRK.A.) From 1964 to 2001, the return on the company’sshares was a staggering 194,936% (for an annualcompound return of 22.6%) versus 4,742% for the S&P500.10 This performance has made Warren Buffett one ofthe most watched investors in the world and the secondrichest person in the United States.11

FYIFYI

FinancialAcct-C09-v3 3/28/06 2:56 PM Page 589

minus total liabilities. But there can be complexities too, such as the legal necessity tokeep track of the sources of funds and keep various kinds of funds separate (for exam-ple, operating funds versus funds donated to the organization for specific purposes), sothese can lead to occasionally complex accounting for equity and related assets andliabilities. There is a whole branch of accounting called fund accounting developed todeal with these complexities: its basic goal is to separate the organization into segmentsthat are accounted for separately in accordance with each segment’s legal or financialsituation, which, therefore, have separate equity accounting, and which frequently haveaccounts receivable from, accounts payable to, and even investments in each other.

Complex Financial Structures

Modern ways of organizing and financing businesses, particularly corporations, havecreated liabilities and equities that are difficult to fit into the double-entry GAAP modelof financial accounting, and that are sometimes difficult to assign as liabilities or equityeven if they are recorded. As you are probably tired of reading, accounting for thesematters is complicated and so is dealt with in advanced accounting books, so this sectionjust describes some of the items to watch out for when reading a corporate balancesheet.

Financing That Is a Mixture of Equity and DebtCorporations’ financing arrangements are often more complex than the simple liabili-ties versus equity categorization traditional in the balance sheet.

• Sometimes a corporation will issue convertible shares or bonds. According to certainrules, and usually at the choice of the holder of the share or bond, such a securitycan be converted to another kind. A preferred share may be convertible to commonshares, or a bond may be convertible to common shares.

• Sometimes a share is redeemable, so that the company has the right to buy it backfrom the holder, or the holder has the right to sell it back to the company, making ita little like debt rather than the permanent capital shares normally are.

• Or term preferred shares may “come due” at a given date, making them also a little likedebt.

These sorts of securities blur the balance sheet’s line between liabilities and equity. If apreferred share is more like a debt than an equity, because, for example, it isredeemable at the option of the holder, GAAP are beginning to require that it beincluded as a liability, not an equity, and dividends paid on such shares would be interestexpense on the income statement, not dividends on the retained earnings statement.

9.5

590 PART THREE DOING FINANCIAL ACCOUNTING

NEL

Here are two questions you should be able to answer basedon what you have just read. If you can’t answer them, itwould be best to reread the material.

1. What are typical differences between an unincorporatedbusiness’s equity and other balance sheet accounting andthat of a corporation?

2. What are shareholder loans, share splits, and stock divi-dends, and what impact do they have on a corporation’sbalance sheet equity section?

how’s your understanding?

Some shares andbonds may beconvertible or haveother characteristicsthat are a mixture ofequity and debt.

FinancialAcct-C09-v3 3/28/06 2:56 PM Page 590

The standard model of financial accounting is clearly creaking under the strain ofsuch “hybrid” securities, and there are increasing views that the balance sheet shouldnot be categorized into debt and equity but rather that the characteristics of eachfinancing source be explained and the decision about how (or if) to categorize it shouldbe left to the user of the financial statements. Considerable information about suchsecurities is already included in the notes, so that the statement user can evaluate theirpresent or potential impact on the figures.

Commitments and OptionsOften, a corporation will make a commitment to issue further shares under certainconditions. Convertible bonds or preferred shares, mentioned above, are such acommitment. There are other types of commitments. A typical example is warrants,which are issued with shares and give the holder the right to buy more shares at a speci-fied price. Another is stock options, which are often awarded to senior managers as a partof their compensation, and again give the holder the right to buy shares at a specifiedprice. This price is expected to be attractive so that the stock option motivates the exec-utive to try to increase the corporation’s share price on the stock market because ahigher share price makes the stock option more valuable. The commitments that haveany likelihood of being honoured are disclosed in the notes, and if issuing the potentialshares could reduce the earnings per share, the potentially lower “diluted” EPS isreported. According to Financial Reporting in Canada 2005, all of the 200 surveyedcompanies reported commitments to issue shares in 2004, with all but one providing adescription of their stock-based compensation plan.

12

Stock options are included in stock-based compensation in accounting standards. Suchoptions, along with employee stock savings plans, provide assistance to employees,management, and directors to own their companies’ shares. Stock options are at thecentre of an extremely controversial accounting problem, one of the most controversialof all. Suppose a company gives the right to buy a share for $5.00 to an employee whenthe fair value of that share reaches a specified value at some future date. Then theemployee is getting a benefit (at least as long as it is realistic to expect that the shareprice will reach that value and that the employee will opt to buy the share). Suppose theexpected value of the benefit is $7 (more about this below). We might expect thearrangement to be recorded something like this:

DR Compensation expense (with Wages) 7CR Compensation liability 7

Then, when the employee buys the share, the share issue could be recorded this way:

DR Cash 5DR Compensation liability 7

CR Share capital 12

But that has not been the approach used. Instead, there has been no recognition of thecompensation, just this entry when the employee buys the shares:

DR Cash 5CR Share capital 5

For a company that has provided generous stock options, this entry fails to record amajor part of the company’s compensation to employees. In particular, it fails to showthe real expense being incurred for such options, and therefore shows a higher incomethan it would if the compensation were deducted as an expense.

Keeping a major compensation off the income statement to make the income meas-ure higher sounds like earnings management to many people (see Section 3.10). In the1990s, with great increases in share prices, stock options were the major form of

CHAPTER 9 LIABILITIES, EQUITY, AND CORPORATE GROUPS 591

NEL

Warrants, stockoptions, and othercommitments to issuemore shares aredisclosed.

FinancialAcct-C09-v3 3/28/06 2:56 PM Page 591

compensation for many companies, especially those in the high-tech, dot-com and tele-com industries. When the accounting standard-setters tried to require the compensationto be recorded as an expense, there was great opposition from many companies. Therewere strong opinions about this, with headlines like “Expensing options is a moveforward,”13 and “Any way you look at it, options are an expense.”14 Perhaps the contro-versy will die down as the great increases in share prices seen in the 1990s have stoppedor been reversed, along with the attractiveness of stock options, but perhaps not!

Some of the opposition to recording the compensation came from companies whowere happy to keep it out of the income calculation, but there are some other reasons:

• The entry above looks simple. Just record the $7 expected benefit given to theemployee. But it is not so simple in practice. • How is the “expected benefit” calculated? The best solution developed so far is to

use a sophisticated options-valuation formula (likely the “Black-Scholes” formula)to calculate what the option is likely to be worth at the date it is granted (assumedto be $7 above), and then update that estimate regularly, adjusting the liabilityand debiting or crediting expense each time an adjustment is needed.

• What if the share price is bouncing around a lot? Is the estimated amount of thebenefit so clear? The formula above takes assumptions about expected share pricevariation into account, but using assumptions makes the value more arguable.

• What if the employee doesn’t buy the share when he or she could, but waits untilthe price is higher than the expected price, or lower? Should such a decision bythe employee affect the compensation expense recorded by the employer?

• What if the share price falls instead, so that when the employee can exercise theoption, the price is $3, so the employee would be better off buying the share onthe open market than by exercising the option (if the employee were then interested in owning any shares at all)?

• What if the company doesn’t do as well as everyone hopes and no or fewemployees ever exercise their options?

In all these cases, any compensation expense is arguable and even hypothetical atany time prior to the employee’s exercising the option, and maybe even then.

Canada became the first major jurisdiction to require the expensing of all stock-basedcompensation awards for all fiscal periods beginning on or after January 1, 2004.

While the concept of recognizing the value of the options granted as employeecompensation expenses is quite simple, though still controversial, the determination ofthe “fair value” can be very complex. The CICA Handbook illustrates the use of the Black-Scholes options pricing model for this purpose. As with some other topics we have intro-duced, the details of this process are best covered in a more advanced course.

Once this value of the options is determined, the accounting is much more straight-forward. As a simplified example let’s assume Options One, a public company, grants toits employees options to purchase 10,000 shares on January 1, 2006. The exercise priceis $25, equal to the market price of the stock on the grant date. All of the options vest atthe end of the three years. Using the Black-Scholes pricing model, the fair value of theoptions is determined to be $90,000. In the absence of a change in the calculated value,the following entry would be made in each year to recognize the compensation cost:

DR Compensation cost 30,000CR Contributed surplus 30,000

Assuming all the options are then exercised on the same day, the entry to record theissuance of stock would be:

DR Cash (10,000 shares x $25) 250,000DR Contributed surplus 90,000

CR Common stock 340,000

592 PART THREE DOING FINANCIAL ACCOUNTING

NEL

FinancialAcct-C09-v3 3/28/06 2:56 PM Page 592

Off-Balance-Sheet Financing and ContractualObligationsNew financial arrangements are being invented all the time, and the impacts they makeon the balance sheet, or might make depending on the company’s accounting policies,are likely to be a factor in their acceptability and popularity. Five examples of financingthat may or may not be well reported in the balance sheet and/or in the notes to thefinancial statements include:

• Ordinary “operating” rental and leasing contracts (capital leases are included in theliabilities, as we saw in Section 8.14). In an article titled “Badly in need of repair,”about alleged severe problems in accounting, The Economist argued in 2002 thatoperating leases make up most off-balance-sheet financing and many should havebeen recorded instead as capital leases, thus getting the assets, but especially thefinancing, onto the balance sheet. The Economist said that in 1997, one-third of theaircraft of the five biggest American airlines were treated as operating leases ratherthan capital leases.15

• The sale of rights to collect accounts receivable so as to speed up the cash inflow inreturn for taking on potential obligations to the party buying those rights.

• Making long-term purchase commitments to get favourable terms.• Making commitments for abnormal expenditures, such as large commitments for

fixed assets.• Financing pensions and other post-employment benefits using trustees. The

trustees’ assets and liabilities do not appear in the company’s balance sheet, asexplained in Section 9.5, but the company is usually required to make up any short-fall in the trustee’s provisions for employee pensions, and may want to take backinto its own income any extra earnings made by the trustee.

• Using subsidiary or associated companies to borrow money so that the commit-ments do not show up on the parent company’s balance sheet. Usually the parentcompany guarantees such debt, so there is a potentially large commitment neverthe-less. This is one of the ways Enron got into trouble: using so-called “special purposeentities (SPEs)” (business partnerships) to handle much of its financing. The rela-tionship between Enron and the hundred or more SPEs it used was not apparent tousers of the company’s financial statements, and they did not appear at all inEnron’s liabilities, though there was some disclosure of them in Enron’s notes.When the SPEs got into trouble, they helped to pull Enron down.16

Many, probably most, of these arrangements are made for good business reasons. Butthere may be a concern among users of financial statements that sometimes suchsources of financing may be sought in order to avoid recording liabilities and so avoidweakening the balance sheet. As these obligations are not included in the financialstatement figures, they can only be learned about by reading the notes, if then. GAAPrequire that significant contractual obligations be disclosed, and Financial Reporting inCanada 2005 indicates that in 2004, 95 of the 200 surveyed companies provided disclo-sures of guarantees other than guarantees of subsidiary debt.

17

Financial InstrumentsThese were mentioned briefly in Section 2.9. The phrase financial instruments refers tothe company’s set of financial assets and liabilities. It also covers various commitmentsand arrangements, such as agreements to purchase currencies in advance to “hedge”against the risk of unfavourable currency fluctuations, “derivative” instruments that arebased on overall market price changes in shares, currencies or commodities, or “swap”contracts in which, for example, an outside bank agrees to pay the interest on some of

CHAPTER 9 LIABILITIES, EQUITY, AND CORPORATE GROUPS 593

NEL

Enterprises may makemany financialcommitments thatare not recorded asliabilities.

Financial instrumentsinclude financial assetsand liabilities, whetheror not recognized inthe accounts.

FinancialAcct-C09-v3 3/28/06 2:56 PM Page 593

the enterprise’s debt if the rate varies beyond some agreed upon range. In its 2004annual report, CPR had about two pages of information about its financial instru-ments.18

• Forward selling or purchasing of U.S. dollars to manage its foreign currency risk(exposure to currency fluctuations). At December 31, 2004, CPR reported thatbased on futures prices then, it had a potential unrealized gain of $0.2 million onthese forward contracts. Neither this gain nor the contracts themselves (aboutU.S.$98.3 million) appeared in the financial statement figures, just in the notes.

• Forward contracts to buy crude oil, to protect against crude oil price fluctuations(CPR uses a lot of fuel). The unrealized gain on the approximately U.S.$146.6million of these oil futures contracts was $32 million at the end of 2004. Again, noneof these were included in the financial statement figures.

• Interest rate swaps, by which someone else agrees to pay the interest on some CPRdebt and CPR in turn pays the other party a different rate. This is done to makeinterest rates more variable, so that the company is not stuck with high-interest debt.At the end of 2004, CPR had swap agreements for a nominal amount of U.S $200million.

• Interest rate risk and fair values of debt. As is explained in Section 10.7, interestrates and market values of bonds and other debts are intimately related. As interestrates rise, debt values tend to fall, because debt paying what had been an attractiveinterest rate becomes less attractive, and vice versa. CPR provided a table of all itsdebts, estimating fair values for each and comparing those to the book values.

• Credit risk on the various financial instruments, arising if the other parties to thevarious arrangements didn’t live up to their side of the bargain. This can happen ifone party makes a bad deal and later cannot, or will not, pay what it has promised.One of the many problems Enron’s collapse created was that it became unable tolive up to billions of dollars of energy contracts, leaving other companies suddenlyliable or unprotected from losses. CPR reported that there was no significant risk ofthis kind from its financial instruments at the end of 2004.

There is literally no end to the kinds of financial instruments that exist, and innova-tions in instruments happen practically daily. Accounting standard-setters have wrestledfor several years with the problem of whether and how to represent these instruments inthe financial statements. Recently, GAAP have required that such instruments bedisclosed in the notes, as we saw with CPR above. In April 2005, the AccountingStandards Board issued three new sections of the Accounting Handbook dealing withfinancial instruments. This new comprehensive guidance largely harmonizes CanadianGAAP with U.S. standards. The new requirements are applicable to fiscal periods on orafter October 1, 2006.

Under the new guidelines, financial instruments are classified as follows:

1. Held for trading investments are designated as such if at the time of purchase theyare acquired principally for the purpose of selling, or they are to be held as part of aportfolio of investments managed together for short-term profit. This classificationis determined primarily by management intent.

2. Held to maturity investments must have fixed or determinable payments (such asthe interest payments on a bond) and a fixed maturity date, and the investing entitymust have a positive intention and the ability to hold the investment to maturity.

3. Loans and receivables are financial assets resulting from the delivery of cash orother assets by a lender to a borrower in return for a promise to repay on a specifieddate or dates, or on demand.

594 PART THREE DOING FINANCIAL ACCOUNTING

NEL

Existence and fairvalues of financialinstruments aregenerally disclosed,but the accounts arenot adjusted.

FinancialAcct-C09-v3 3/28/06 2:56 PM Page 594

Corporate Groups: IntercorporateInvestments

Modern businesses, especially large ones, are often groups of separately incorporatedcompanies. Such corporate groups were discussed in Section 2.9. This section and the nextexpand on that discussion, describing consolidated financial statements for corporategroups but also some other accounting issues involved in such groups. Accounting forcorporate groups is a complicated part of financial accounting and is covered in detailin the financial accounting standards of the CICA and FASB. This book introduces youto the main principles behind it and shows you how to apply the principles to do somebasic calculations.

Kinds of Intercorporate InvestmentsCorporations invest in other companies in many ways and for a variety of reasons. Theinvestment may be passive in nature, or made for one of the following reasons:

1. An investment may be targeted to earn a higher return on idle cash than thatprovided by bank accounts. Typically of low risk and readily convertible to cash,these investments may generate income from interest or dividends, or capital gainsif the price rises.

2. An actively managed portfolio of investments may be used in the normal course ofbusiness to earn returns on price changes. Such active trading investment portfoliosare most often used by financial institutions who may earn further returns by actingas a dealer.

3. Cash may also be invested in longer-term securities to earn higher rates of return.While such investments may be made in equity securities, money market instru-ments such as bonds are more commonly used. The intent is often to hold suchdebt instruments to maturity.

Corporations also make strategic investments “in equity instruments issued byanother entity, with the intention of establishing or maintaining a long-term operatingrelationship with the entity in which the investment is made.”19 Such investments maybe accounted for as subsidiaries, investments subject to significant influence, interests injoint ventures.

Six common investments are summarized in Exhibit 9.1 and will be examined inthis section and the next.

9.6

CHAPTER 9 LIABILITIES, EQUITY, AND CORPORATE GROUPS 595

NEL

Many businesses arereally groups ofcorporations.

Here are two questions you should be able to answer basedon what you have just read. If you can’t answer them, itwould be best to reread the material.

1. Why do redeemable or convertible securities issued by acorporation cause some difficulties for the traditionalbalance sheet format?

2. Why is off-balance-sheet financing a controversial problemfor accounting?

how’s your understanding?

4. Available-for-sale financial assets are those financial assets that are designated asavailable for sale or that are not classified as one of the three categories describedabove.

FinancialAcct-C09-v3 3/28/06 2:56 PM Page 595

Figure 9.1 illustrates a corporate group, with Corporation A at its financial centre.This section will review how to account for A’s investment in Corporations B, D, and H,none of which A controls, and the next section will cover accounting for A’s investmentin C, E, F, and G, all of which A controls.

1. Temporary Investments (Short-Term Passive Investments)An investment is classified as a “temporary investment” if it matures within one year or if itcan be liquidated reasonably quickly and the investment is intended to be a temporary useof idle cash. Held-for-trading investments would usually be temporary in nature. Held-to-maturity investments would be classified as long-term unless the maturity falls within oneyear. Available-for-sale investments may be either current or long-term, the determinationof which requires careful consideration of the conditions described above.

The potentially significant effect on the reported liquidity position of the entitymakes proper determination of the current vs. long-term classification of these invest-ments critical.

2. Long-Term Passive InvestmentsAs noted above, held-to-maturity investments will be long-term unless the maturity fallswithin one year and under certain circumstances available-for-sale investments may alsobe classified as long-term. Held-to-maturity investments are, by definition, debt securities.While management’s intent is to hold these securities to maturity, they may be sold at anytime. Available-for-sale and held for trading investments may be either debt securities orshare investments.

If the investment is in voting shares then it may be classified as either passive orstrategic in nature depending on the intention behind and control involved in the invest-ing corporation’s (investor’s) ownership of the investment.

3. Long-Term Strategic InvestmentIn a strategic investment, the investor owns enough of the investee company to influ-ence what it does, but may not entirely control it. A strategic investment in commonshares may be classified as a significant influence investment, a control investment, or ajoint venture. The investing corporation is said to exercise significant influence if it hasmore than the 20% voting interest suggested above but not voting control (not more

596 PART THREE DOING FINANCIAL ACCOUNTING

NEL

Temporaryinvestments are themost short-term andpassive intercorporateinvestments.

In a passiveinvestment, theinvestor does notexercise significantinfluence over theinvestee.

The equity basis isused when theinvestor exercisessignificant influenceover the investee.

9•1EX

HIBIT

Classification and Accounting for Investments

Classification Investment Type Accounting Method

Passive Hold-to-maturity Cost

Passive Available for sale Fair value: gains & losses are recognized in other comprehensiveincome until sold

Passive Trading investment Fair value: gains & losses arerecognized in net income

Strategic Control investment Consolidation

Strategic Significant influence Equity method

Strategic Joint venture Proportionate consolidation

FinancialAcct-C09-v3 3/28/06 2:56 PM Page 596

than 50%). This is illustrated in Figure 9.1 as A’s investment in D. In this case, the equitybasis of accounting is used. Under this basis, the investing corporation includes in itsincome statement and balance sheet its share of earnings by the investee company,because it has influenced that company’s performance and can take some credit for it.

Under the equity basis,

• the investor’s investment asset is still valued initially at cost, as it was for passiveinvestments using the cost basis.

• As the investee corporation earns income (or incurs losses), the investor corporation’sasset is increased for the investor’s share of that income (or decreased for its shareof losses!) and that share is included in the investor’s income. This is an accrual ofincome the investor is entitled to. The investor is taking credit for its share of theinvestee’s income (increase in retained earnings). Nonoperating “other” revenue iscredited with this share and the investment asset account is debited, so that assetaccount is treated like an account receivable for the accrued income.

• When the investee pays a dividend, the investor receives some of the accruedincome as its share of the dividend, so the dividend received is deducted from theinvestor’s investment asset account, just as collection of an account receivable wouldbe deducted from the account receivable asset. The dividend is not called incomeby the investor because the income has already been accrued; instead, the dividend isdeducted from the investment asset because it is considered a return of some of themoney invested.

• There are some other more complicated features of equity basis accounting we willnot get into.

Financial Reporting in Canada 2005 reports that in 2004, 124 of the 200 companiessurveyed had long-term investments other than joint ventures (mentioned at the end ofthis section). Of these, 24 used the cost basis, 19 used the equity basis, 72 used both, 1found some other way of accounting for their investment, and 8 did not say what basisthey used.

20

A Cost and Equity Basis ExampleExhibit 9.2 summarizes how the two methods work (ignoring complexities):

Here is an example. Grand Ltd. acquired investments in two other corporations onJanuary 1, 2006. These acquisitions are (a) and (b) in the list below; events (c) to (e)also took place in 2006.

a. 60,000 shares (15% of the voting interest) in AA Ltd. were purchased for $1,800,000cash. Because this was to be a fairly passive long-term investment, Grand wouldaccount for it using the cost basis.

b. 145,000 shares (29% of the voting interest) in BB Ltd. were purchased for$4,640,000 cash. Since Grand intended to participate in the management of BBLtd., Grand would account for it using the equity basis.

c. On June 30, both investees announced their earnings per share for the first sixmonths of 2006: $2 per share for AA Ltd. and $2.10 per share for BB Ltd.

d. On December 10, both investees paid dividends to shareholders: $1.50 per share forAA Ltd. and $1.60 per share for BB Ltd.

e. On December 31, both investees announced their earnings per share for 2006:$3.40 per share ($1.40 additional since June 30) for AA Ltd. and $3.90 per share($1.80 additional since June 30) for BB Ltd.

CHAPTER 9 LIABILITIES, EQUITY, AND CORPORATE GROUPS 597

NEL

The equity basis isaccrual accounting,the investor’s share ofthe investee’s incomeis accrued.

When the investorreceives a dividendfrom the investee,that is collection ofaccrued income.

FinancialAcct-C09-v3 3/28/06 2:56 PM Page 597

598 PART THREE DOING FINANCIAL ACCOUNTING

NEL

9•2

EXHIBIT

Cost basis Equity basis

Initial carrying value of the investor’s Original cost Original costintercorporate investment asset

Investor’s share of income earned Nothing done Add to asset and toby investee other revenue

Investor’s share of dividend paid Add to cash and Add to cash and deductby investee to other revenue from investment asset

Resulting balance sheet value of Just original cost Original cost plus accruedthe investor’s intercorporate income share minus shareinvestment asset of dividends paid

A

E

F

G

H

B C D

Corporation A is the main company in Group A, an economic entity greater than Aby itself and encompassing the separate corporate legal entities.

A owns all of the voting shares of C and E: they are entirely part of Group A andso are combined with A in A’s consolidated financial statements.

A owns more than 50% of the voting shares of F and G, so they are consolidatedtoo, but some accounting has to be made for the parts of them A does not own.

A does not control D but has significant influence over it, so it will be accounted for as a long-term active investment (equity basis).

A has no significant portion of the voting shares of B or H, so they will be accounted for as passive investments, either short-term or long-term dependingon A’s intent.

Joint ventures and mergers are not illustrated here.

Group A

figure 9.1

Illustration of aCorporate Group

FinancialAcct-C09-v3 3/28/06 2:56 PM Page 598

The effects of items (a) to (e) on the financial statements of Grand Ltd. at the end of2006 are:

INVESTMENT IN AA LTD. (COST BASIS)a. Long-term investment asset starts out at the purchase price of $1,800,000 (cash

reduced by same amount).b. (Concerns only BB Ltd.)c. Earnings announcement is ignored for accounting purposes in the cost basis.d. Cash received and dividend revenue are recorded for $90,000 (60,000 shares ×

$1.50).e. Earnings announcement is ignored for accounting purposes in the cost basis.

Using the cost basis, Grand Ltd.’s financial statements as of December 31, 2006, willtherefore include for AA Ltd.:

Grand Ltd. balance sheet at end of 2006Investment in AA Ltd. (noncurrent asset) $1,800,000

Grand Ltd. income statement for 2006Dividend revenue (in other revenue) $ 90,000

INVESTMENT IN BB LTD. (EQUITY BASIS)a. (Concerns only AA Ltd.)b. Long-term investment asset starts out at the purchase price of $4,640,000, the same

as if the cost basis were used.c. Upon earnings announcement, both investment revenue and investment asset are

increased by $304,500 (145,000 shares × $2.10), Grand’s share of BB’s income.d. Cash is increased by and investment asset is reduced by $232,000 (145,000 shares ×

$1.60): the dividend received is therefore deemed to be a return to Grand Ltd. ofsome of its investment.

e. Upon earnings announcement, both investment revenue and investment asset areincreased by $261,000 (145,000 shares × $1.80), Grand’s share of BB’s income.

Using the equity basis, Grand Ltd.’s financial statements as of December 31, 2006, willtherefore include for BB Ltd.:

Grand Ltd. balance sheet at end of 2006Investment in BB Ltd. (noncurrent asset) $4,973,500

Grand Ltd. income statement for 2006Investment revenue (in other revenue) $ 565,500

Calculations:$4,973,000 = $4,640,000 + $304,500 – $232,000 + $261,000$565,500 = $304,500 + $261,000

4. Joint VentureA joint venture is a partnership between the investing corporation and other investors,usually formed to conduct exploration (such as in the oil and gas industry), develop newproducts, or pool resources in some other way. It is common in international business.Here, the investing corporation does not have control, but it does have significant influ-ence on the joint venture because of the partnership arrangement and the businessarrangements among the joint venture and the partner investors.

Joint ventures are accounted for in the investing company’s financial statementsusing “proportionate consolidation,” in which the investing company’s proportionateshares of the venture’s assets and liabilities (say, 50% if the venture is 50% owned) arecombined with the investing company’s assets and liabilities. We will not examine thismethod in this book.

CHAPTER 9 LIABILITIES, EQUITY, AND CORPORATE GROUPS 599

NEL

Equity basis: asset andrevenue increased forincome share; assetdecreased upondividend.

Cost basis: no changein asset; dividendadded to revenuewhen received.

FinancialAcct-C09-v3 3/28/06 2:56 PM Page 599

Corporate Groups: ConsolidationThis section considers the remaining kind of investment summarized in Exhibit 9.1,combining the financial statements of a group of corporations into one set of consolidated financial statements representing the group.

5. Acquisition by One Corporation of AnotherFrequently, one corporation acquires more than 50% of the voting shares of another,becoming the majority owner of the other. This was illustrated in Figure 9.1 as A’s invest-ment in C, E, F, and G. This is done for many reasons, including to operate the twocompanies jointly and gain the benefits of such coordination. As long as voting controlis held, the investing company can gain many benefits without having to own all theshares of the other, though it may well do so. Financial accounting uses a techniquecalled consolidation to present the two companies as one economic entity, almost as if theywere one company. As noted in Figure 9.1, the economic entity is greater than any ofthe individual legal entities represented by the corporations in the group.

Until recently, a business combination could be defined as a merger, a coming togetherof equals, instead of an acquisition by one of the other. To summarize a long and loudcontroversy, accounting standard-setters finally concluded that the accounting methodfor mergers, called “pooling of interests,” led to too many abuses and masked what wasnearly always one company dominating the other. Therefore, all corporate combina-tions are accounted for using a method called the “purchase method” to distinguish itfrom pooling of interests. The company that is in control is also called the parent andthe controlled company is called the subsidiary.

Consolidation is imaginary: there is no legal, consolidated entity. Rather, it is legally agroup of separate companies with connected ownership. The idea is to present thegroup of companies as if it were a single entity. This method is thought to represent theeconomic and business circumstances more faithfully than would reporting separatestatements for all the legally separate companies and leaving the user to try to add themtogether.

Consolidation uses a simple idea: to prepare the financial statements of a group ofcompanies, put the balance sheets, income statements, and other statements for all thecompanies side by side and, mostly, add them up. The consolidated balance sheet’s cashfigure would be the sum of all of the companies’ cash figures, the consolidated incomestatement’s cost of goods sold expense figure would be the sum of their cost of goods

9.7

600 PART THREE DOING FINANCIAL ACCOUNTING

NEL

Consolidationrepresents thecorporate group asone economic entity.

A corporatecombination isaccounted for as theacquisition of onecorporation by theother.

Consolidationportrays separatecorporations as if theywere a single entity.

The basic idea ofconsolidation is toadd the companies’accounts together.

Here are two questions you should be able to answer basedon what you have just read. If you can’t answer them, itwould be best to reread the material.

1. If Gretel Ltd. buys a noncontrolling number of shares ofHansel Inc. for $460,000, what are the criteria by whichmanagement of Gretel should decide if the investment isto be accounted for on the cost basis or the equity basis?

2. During the year, Gretel receives a $45,000 dividend fromHansel. At the end of the year, Hansel reports a netincome. If Gretel’s proportion of the Hansel voting sharesis applied to Hansel’s net income, the resulting figure is$78,500. What income from its investment in Hansel willGretel report if it is using the cost basis? The equity basis?

What is the “Investment in Hansel” asset on Gretel’s booksat the end of the year on the cost basis? The equity basis?

Cost basis income = $45,000, the dividend;

Equity basis income = $78,500 (Gretel’s share of Hansel’sincome);

Cost basis asset = $460,000 (no change from the investment inpart 1);

Equity basis asset = $493,500 ($460,000 + $78,500 share ofincome – $45,000 dividend).

how’s your understanding?

FinancialAcct-C09-v3 3/28/06 2:56 PM Page 600

sold figures, and so on. But accounting is never that simple, is it? To apply this simpleidea to the complexities of modern businesses, a quite complicated set of GAAP forconsolidation has arisen. In this book, the complexities will be left out in favour of afocus on three main issues in consolidation accounting:

1. What to do if the parent company owns less than 100% of the subsidiary’s votingshares (Corporations F and G in Figure 9.1).

2. Determining the asset and liability values that are to be added together. 3. Determining any “goodwill” arising from the acquisition price paid by the parent.

Three Basic Concepts in Purchase MethodConsolidation1. Noncontrolling (minority) interest. This arises if the parent owns less than 100% of the

subsidiary, and equals the percentage of the voting shares not owned by the parenttimes the subsidiary’s shareholders’ equity at the date of acquisition, adjusted forchanges since that date. • For example, if P Inc. bought 75% of the voting shares of S Ltd. on January 3,

2003, when S Ltd.’s shareholders’ equity equalled $300,000, then the noncontrol-ling interest, which many consolidated financial statements refer to as the minor-ity interest, would equal 25% of that, or $75,000.

• This amount is shown as a liability on the consolidated balance sheet. The liabil-ity represents the part of the joint consolidated entity’s equity not owned by theparent company’s shareholders. It represents someone else’s equity in the group, so it isnot included with the consolidated equity. It is not a debt of the consolidated entity,because it need not be paid—you could think of it as an acknowledgment thatthe consolidated entity has an obligation to the minority owners of S Ltd., whodid not sell their shares to P Inc.

• So the noncontrolling interest is another of the accounts that blurs the distinc-tion between liabilities and equity.

• Over time, the noncontrolling interest liability is accounted for similarly to theway the investment asset was in the equity method in Section 9.6:• The liability is increased (and consolidated net income is decreased) each year by

the minority owners’ share of the subsidiary’s net income; • It is decreased (and consolidated cash is decreased) whenever the minority

owners receive a dividend. • So if the parent company succeeds in getting the subsidiary to earn income,

some of that income is credited to the minority owners, and when theminority owners are paid a dividend, the liability is reduced just as anyliability would be.

2. Balance sheet asset and liability values. The idea of consolidation is just to add theaccounts together: the parent’s accounts receivable are added to the subsidiary’saccounts receivable, the land is added to the land, the accounts payable are addedto the accounts payable, the revenue is added to the revenue, the income taxexpense is added to the income tax expense, and so on. But four exceptions tosimply adding the parent’s and subsidiary’s balance sheets together are important inunderstanding how consolidation works:

(1) Any intercompany balances are ignored. If S Ltd. owes P Inc. $40,000, for example,that would be on S Ltd.’s balance sheet as an account payable and on P Inc.’sbalance sheet as an account receivable. If the consolidated balance sheet is torepresent the two companies as if they were one entity, then this $40,000 amountis an internal matter to that entity: it is not owed to or receivable from anyoneoutside the entity, so it is not like the other accounts payable and accounts receiv-

CHAPTER 9 LIABILITIES, EQUITY, AND CORPORATE GROUPS 601

NEL

Minority’s sharedecreasesconsolidated incomeand increases liability.

Consolidation startsby adding together allthe parent’s andsubsidiary’s accounts.

Noncontrolling(minority) interestliability is the part ofthe entity the parentdoes not own.

FinancialAcct-C09-v3 3/28/06 2:56 PM Page 601

formula

able. Therefore, it is just left out of the consolidated figures. Intercompany salesand expenses, such as management fees, are also left out of the income state-ment, and any profit made by one company in dealing with the other is left outas well. Eliminating these can be complex: we won’t deal with them.

(2) The account for the parent company’s investment in the subsidiary is also anintercompany account, so it is ignored in the consolidation.

(3) Another intercompany amount left out is the parent’s share of the subsidiary’sshareholders’ equity. It is what the parent bought, so it is included in theparent’s investment account and therefore is not part of the consolidated equityexternal to the consolidated entity. The part of the subsidiary’s equity notbought is transferred to the noncontrolling interest liability (as noted above), sothe result is that none of the subsidiary’s equity at the date of acquisition isincluded in consolidated equity. Consolidated equity at date of acquisition equals justthe parent’s equity alone. The parent purchased shares of the subsidiary, and via itsvoting interest, its owners control the assets and liabilities of the subsidiary. Thesubsidiary’s assets and liabilities are therefore included in the consolidatedbalance sheet, and to avoid double counting, the intercompany accounts forparent’s investment and subsidiary’s equity are eliminated.

(4) The last exception to just adding the companies’ accounts together is to recog-nize that, when it acquired the subsidiary, the parent company may have haddifferent values in mind for various of the subsidiary’s assets and liabilities thanthe amounts shown for those on the subsidiary’s balance sheet. Because a trans-action did happen (the parent bought shares of the subsidiary), the historicalcost basis of accounting requires that any revised values at that date be takeninto account. These are called the fair values at that date; they can be viewed asthe cost to the parent of the subsidiary’s net assets, and there was a transactionto give them validity (the purchase of the subsidiary’s shares). However, becausethe minority owners did not sell, there was not a transaction for their share ofthe subsidiary’s assets and liabilities, so the minority’s share is not taken into accountin revaluing the subsidiary’s assets and liabilities to fair values. We know what theparent paid for what it got; we do not know what might have been paid for whatit did not buy (the minority’s share). Therefore, at the date of acquisition, each ofthe subsidiary’s assets and liabilities is added into the consolidated figures usingthe following formula:

Amount used in Book value Parent’s (Fair value –consolidation = in subsidiary’s + share of × subsidiary’s

calculation balance sheet subsidiary book value)

Example of Fair Value Adjustment

• Subsidiary’s land was on its balance sheet at a value (presumably cost) of$120,000.

• Parent’s evaluation indicated its fair value was $180,000.• Parent bought 85% of the subsidiary.• Therefore, using the formula above, the land would be included in the consoli-

dated figures at a value of $171,000 [$120,000 + 0.85 ($180,000 – $120,000)]. • The last term is the adjustment to reflect the parent’s cost for the 85% interest it

acquired, maintaining the historical cost basis for the consolidated financialstatements. Therefore, the consolidated figures do not fully revalue thesubsidiary’s assets and liabilities: the minority owners’ share of such revaluationis left out because they did not sell.

602 PART THREE DOING FINANCIAL ACCOUNTING

NEL

At the acquisitiondate, the subsidiary’sassets and liabilitiesare revalued only forthe parent’s share.

At acquisition,consolidatedshareholders’ equityequals just theparent’s equity.

FinancialAcct-C09-v3 3/28/06 2:56 PM Page 602

formula

Any revaluations of assets and liabilities that are done may affect futureconsolidated income; for example, if the subsidiary’s buildings and equipmentare increased in value in the consolidation, then the consolidated amortizationexpense will have to be increased too, in order to take that into account. This isanother complication we will not take any further!

3. Goodwill arising on consolidation. What if P Inc. paid more for the shares of S Ltd.than the sum of the fair values of S Ltd.’s assets minus its liabilities? This indicatesthat P Inc. is buying something else not on S Ltd.’s balance sheet, something in addi-tion to all the individual parts of S Ltd. This something is called goodwill, or goodwillarising on consolidation. As explained in Section 8.14, it might represent goodmanagers, a good location, faithful customers, economies of scale with the parent,reduced competition, or other factors the parent company took into account inagreeing to a price for the subsidiary’s shares:

Consolidated Cost of parent’s investment –goodwill = Parent’s portion of (Fair values of subsidiary’sasset assets – Fair values of its liabilities)

Example of Consolidated Goodwill Calculation

• Very Big Inc. paid $1,200,000 for 80% of the voting shares of Not So Big Ltd.• At that date Very Big evaluated Not So Big’s assets to be worth $4,300,000 and its

liabilities to be $3,000,000.• Using the above formula, consolidated goodwill at the date of acquisition would

be $160,000 [$1,200,000 – 0.80 × ($4,300,000 – $3,000,000)].

Figure 9.2 illustrates the three valuations of the subsidiary company. The first layer isthe company’s original book value. Since the parent has less than 100% control ofthe subsidiary, a noncontrolling (minority) interest is recorded as a liability on theparent’s balance sheet. The minority interest liability is based on the original bookvalue of the subsidiary. The second layer is the fair value of the subsidiary, assessedby the parent when the parent purchases the subsidiary’s shares. Finally, if the thirdlayer (the purchase price) is greater than the fair value of the company, goodwill iscreated. Note, the fair values and goodwill portions are only calculated based uponthe share the parent owns of the subsidiary (e.g. 80% in the prior example).

Consolidated goodwill is shown among the noncurrent assets on the consolidatedbalance sheet. As noted in Section 8.14, it used to be amortized over time but is nowmaintained at its historical value unless an annual review of it indicates that its value hasbeen impaired. In that case, it is written down to its estimated fair value and the write-down is included as an expense for the year (probably separated from operatingexpenses).

Two wrinkles regarding goodwill might as well be mentioned.

(1) If the difference is negative (investment cost is less than the parent’s portion of thenet sum of the fair values), you might expect this to be called “badwill” and to beshown on the consolidated balance sheet too. But under GAAP, it is assumed thatthere was something wrong with the subsidiary’s assets for this to happen, so the fairvalues are reduced in the consolidation calculation until the parent’s portionexactly equals the purchase price. The result is that goodwill (or badwill) is zero.

(2) If the subsidiary already had goodwill, that is wrapped into the new goodwill figureand not carried forward separately.

CHAPTER 9 LIABILITIES, EQUITY, AND CORPORATE GROUPS 603

NEL

Consolidated goodwillequals parent’sinvestment cost minuspartially revaluedsubsidiary.

Goodwill appears onlyif it is an asset (apositive differencebetween cost and fairvalues).

FinancialAcct-C09-v3 3/28/06 2:56 PM Page 603

To summarize, the consolidated balance sheet at date of acquisition includes

• the parent company’s balance sheet figures;• the subsidiary’s assets and liabilities, revalued to reflect the parent’s portion of any

increases and decreases to fair values;• any noncontrolling (minority) interest in the subsidiary’s equity; and• any consolidated goodwill.

The latter two, minority interest and goodwill, do not appear on either of the individualcompanies’ balance sheets—they arise only when the corporate group is consolidated.The revaluations of the subsidiary’s assets and liabilities also appear only in the consolidated balance sheet—the subsidiary’s balance sheet continues with the values ithad prior to acquisition, except that usually the subsidiary’s accounting policies arechanged to match the parent’s and that can result in changes in the allowance fordoubtful accounts, inventory valuation, accumulated amortization, and so on.

The consolidated balance sheet does not include• the parent’s account for investment in subsidiary;• the subsidiary’s shareholders’ equity; or• any other intercompany asset and liability accounts.

An Example of Consolidation at Date of AcquisitionABC Company purchased 80% of XYZ Company’s voting shares for $500,000. We havethe following information for XYZ as at the date of acquisition (no intercompany receivables or payables existed at that date):

604 PART THREE DOING FINANCIAL ACCOUNTING

NEL

Portion of subsidiary acquiredby the parent company

Portion of subsidiary NOT acquiredby the parent company

Goodwill

Ignored in theconsolidated figures:only the book valueof the minorityinterest is included

Noncontrolling(minority) interest

Fair Value –Book Value

Price Paid

Fair Value

Subsidiary’sBook Value

When a company is purchased, there are different layers reflectingthe valuation of the company.

figure 9.2

FinancialAcct-C09-v3 3/28/06 2:56 PM Page 604

CHAPTER 9 LIABILITIES, EQUITY, AND CORPORATE GROUPS 605

NEL

9•4

EXHIBIT

IncludeGoodwill

Balance Sheet Book Adjust 80% and Non- ConsolidatedValues as at Date of FV – BV* controlling Balance

of Acquisition of XYZ Interest Sheet

ABC XYZ

Cash $ 175,000 $ 45,000 $ 0 $ 220,000Receivables 425,000 75,000 (12,000) 488,000Inventory 660,000 100,000 16,000 776,000Investment in XYZ 500,000 — Ignore —Property & equipment 1,700,000 200,000 80,000 1,980,000Consolidated goodwill — — $156,000 156,000

$3,460,000 $420,000 $3,620,000

Payables $ 730,000 $ 90,000 $ 4,000 $ 824,000Long-term debt 850,000 0 0 850,000Consolidated noncontrolling interest — — $ 66,000 66,000

Common shares 100,000 50,000 ABC only 100,000Retained earnings 1,780,000 280,000 ABC only 1,780,000

$3,460,000 $420,000 $3,620,000

* FV – BV = Item’s fair value – Its value on XYZ’s balance sheet.

a. Goodwill arising on consolidation. ABC paid $500,000 for 80% of $430,000 (the identifiable fair values):

Purchase price $500,000Minus acquired fair value (80% × $430,000) 344,000

Goodwill $156,000

b. Noncontrolling interest. ABC purchased only 80% of XYZ. Therefore, the other 20% isthe owners’ equity of the noncontrolling owners. It is 20% of the book value ofXYZ’s owners’ equity at the date of acquisition, or 20% × ($50,000 + $280,000) =$66,000.

c. Consolidated figures for ABC are shown in Exhibit 9.4.

9•3

EXHIBIT

XYZ Data Book Values Fair Values

Cash $ 45,000 $ 45,000Accounts receivable 75,000 60,000Inventory 100,000 120,000Property and equipment (net) 200,000 300,000

$420,000 $525,000

Accounts payable $ 90,000 95,000Common shares 50,000Retained earnings 280,000

$420,000

Sum of fair values of net assets $430,000

FinancialAcct-C09-v3 3/28/06 2:56 PM Page 605

A Comment on Consolidated Net Income AfterAcquisitionConsolidated net income is the sum of the incomes earned since acquisition, withadjustments to remove intercompany balances and any subsidiary’s noncontrollingowners’ interest in the income earned by the subsidiary and to reflect amortization ofgoodwill, among other things. The calculation is as follows:

Start with the sum of the parent’s and the subsidiaries’ incomes $ XXXX

Subtract:

a. Any profits earned by any of the companies on intercompany sales. (XXXX)

b. Any income from the subsidiaries already included in the parent’sor other subsidiaries’ accounts through use of the equity method of accounting on the companies’ individual financial statements (these are intercompany amounts too). (XXXX)

c. Any extra amortization and other expenses resulting from adjusting subsidiaries’ assets and liabilities to fair values in the consolidation. (XXXX)

d. Any noncontrolling owners’ share of the net income earned by the subsidiary of which they remain part owners (roughly equal to the noncontrolling ownership percentage multiplied by the subsidiary’s net income). (XXXX)

e. Write-down of any impaired goodwill arising on consolidation. (XXXX)

The result is consolidated net income $ XXXX

You can see that consolidated net income is likely to be less than the sum of the individ-ual companies’ net incomes, perhaps substantially less if goodwill impairment is large orthere are significant minority interests.

606 PART THREE DOING FINANCIAL ACCOUNTING

NEL

Here are two questions you should be able to answer basedon what you have just read. If you can’t answer them, itwould be best to reread the material.

1. On January 1, 2003, Supersix Inc. bought 75% of thevoting shares of Weaknees Ltd. for $231,000 cash. At thatdate, Weaknees’s balance sheet showed assets of$784,000 and liabilities of $697,000. Supersix assessedthe fair values of Weaknees’s assets to be $800,000 and itsliabilities to be $690,000 at acquisition date. Why dononcontrolling (minority) interest and goodwill appear onSupersix’s consolidated balance sheet, and what were thefigures for those items at acquisition date?

MI appears because it represents the equity that the parent didnot acquire.

MI = 25% of ($784,000 – $697,000) = $21,750.

GW appears because it is part of the acquisition cost: forunidentifiable but valued assets.

GW = $231,000 – 75% of ($800,000 – $690,000) = $148,500

2. At the same date, Supersix’s balance sheet showed assetsof $56,782,000 and liabilities of $45,329,000. Whatwould be the consolidated equity of Supersix after consolidating Weaknees?

Consolidated equity is just Supersix’s ($56,782,000 –$45,329,000 = $11,453,000).

how’s your understanding?

Consolidated netincome is reduced byextra amortization,etc.

FinancialAcct-C09-v3 3/28/06 2:56 PM Page 606

Terms to Be Sure You UnderstandHere is this chapter’s list of terms introduced or emphasized. Make sure you know what they mean in accounting, andif any are unclear to you, check the chapter again or refer to the glossary of terms at the back of the book.

9.8

CHAPTER 9 LIABILITIES, EQUITY, AND CORPORATE GROUPS 607

NEL

Accounting policiesAccumulated foreign currency

translation adjustmentAcquisitionBondBonded debtBusiness combinationsCallable debtCapitalCapital cost allowanceCapital leasesConditional sale contractConsolidatedConsolidated financial statementsConsolidationContingent assetsContingent liabilitiesContra accountContributed surplusConvertibleCorporate groupsCorporationCost basisCurrent portion of income tax

expenseDebentureDeferred income tax

Demand loansDiscount Earnings managementEconomic entityEffective income tax rateEquityEquity basisFair valuesFinancial instrumentsFund accountingFuture income tax liabilityFuture income taxesFuture portion of income tax

expenseGoing concernGoodwillGoodwill arising on consolidationHistorical costIncome tax payableIndentureInterperiod tax allocationIntraperiod tax allocationJoint ventureLiabilitiesLoans from shareholdersMatching principleMerger

Minority interestMortgageNoncontrolling interestParentPartnership(s)Personal guaranteesPremiumPresent valueProprietorship(s)ProvisionsPurchase methodRedeemableSecurityShare splitsSignificant influenceStock-based compensationStock dividendsStock optionsSubsidiaryTemporary difference(s)Temporary investmentsTreasury sharesWarrantsWorking capitalWorking capital ratio

FinancialAcct-C09-v3 3/28/06 2:56 PM Page 607

9.9

Installment 9

CONTINUING DEMONSTRATION CASE

NEL608

In this installment, we’ll prepare the complete first year’s financial statements and notesfor the company. The statements will be analyzed in Installment 10, so make sure youare clear about how they are assembled below.

DATA FOR INSTALLMENT 9

In Installment 7, the February 28, 2007, adjusted trial balance of Mato Inc. wasprepared, and in Installment 8, the company chose its accounting policies. The data aretherefore the trial balance and the policies. Refer back to those installments if you areunsure about any results below.

RESULTS FOR INSTALLMENT 9

With some accounting help, Tomas prepared the set of financial statements and notesfor the company’s first year.

Mato Inc.Statement of Income and Deficit

for the Year Ended February 28, 2007

Revenue $229,387Cost of goods sold 138,767

Gross profit $ 90,620Operating expenses:

Bad debts $ 2,436Salaries 67,480Travel 9,477Telephone 4,014Rent 24,000Utilities 3,585Office and general 5,688Inventory shortage 441Interest 6,469Amortization 21,096 144,686

Net loss for the year (no tax) $ (54,066)Retained earnings, March 1, 2006 0

Deficit as at February 28, 2007 $ (54,066)

FinancialAcct-C09-v3 3/28/06 2:56 PM Page 608

NEL 609

Mato Inc.Cash Flow Statement

For the Year Ended February 28, 2007

Operations:Net loss for the year $ (54,066)Add back amortization for the year 21,096 Changes in noncash working capital accounts:

Increase in accounts receivable $ (14,129)Increase in inventory (33,612)Increase in prepaid expenses (525)Increase in accounts payable 40,708Increase in deferred revenue 500 (7,058)

Cash used in operations $ (40,028)Investing activities:

Equipment, leasehold improvements, and software acquired (116,054)

Financing activities:Bank borrowing $ 47,500Repayment of loan (15,000) 32,500

Decrease in cash during the year $(123,582)Cash on hand, March 1, 2006 130,000

Cash on hand, February 28, 2007 $ 6,418

Mato Inc.Balance Sheets

at February 28, 2007 and March 1, 2006

Assets Liabilities and Equity

2007 2006 2007 2006

Current assets: Current liabilities:Cash $ 6,418 $130,000 Bank loan $ 47,500 $ 0Receivables (net) 14,129 0 Payables 41,808 1,100Inventory 33,612 0 Loan payable 0 15,000Prepaid expense 525 0 Deferred revenue 500 0

$ 54,684 $130,000 $ 89,808 $ 16,100

Noncurrent assets: Shareholders’ equity:Equipment cost $ 57,290 $ 10,000 Share capital $125,000 $125,000Accum. amort. (7,344) 0 Deficit (54,066) 0Leasehold (net) 51,172 0 $ 70,934 $125,000Software (net) 3,840 0Incorp. cost 1,100 1,100

$106,058 $ 11,100

TOTAL $160,742 $141,100 TOTAL $160,742 $141,100

FinancialAcct-C09-v3 3/28/06 2:56 PM Page 609

NEL610

DISCUSSION OF THE RESULTS

The results for the year were still negative: a loss of $54,066 and a decrease in cash of$123,582. However, there was quite an improvement compared with the first six months:

• The loss for the first six months (Installment 3) had been $49,378, so the additionalloss for the second six months was relatively small at only $4,688.

• The cash decrease for the first six months (Installment 4) was $125,493, so there wasan addition to cash of $1,911 during the second six months.

• The working capital at the end of August (Installment 3) was negative at $38,772($96,844 – $135,616) and by February 28, 2007, was still negative at $35,124($54,684 – $89,808), but a little less negative.

Further analysis of the results will be conducted in Installment 10. However, Mavis andTomas wonder if the financial statements have made their company’s performanceappear worse than necessary and if some other choice of accounting policies mightmake things look more optimistic.

Mavis and Tomas’s concern regarding changing their accounting policies to makethings look rosier is understandable. They have worked hard to make their companysucceed, but the first year’s results are not positive. If they had paid themselves nosalaries for their year’s work, the company would have shown a tiny income ($54,280salaries per note 5 above minus the $54,066 loss would equal $214 income beforeincome tax). But that would have been misleading because it would fail to measure thevalue added by their efforts—not to mention that they would have starved! Would itmake sense to try to find accounting policies that would improve the picture? Theanswer is no, for the following reasons:

1. Such manipulation would be ethically questionable and perhaps even dangerous, ifit obscured the company’s real problems and reduced the pressure on Mavis andTomas to improve Mato’s performance. They may feel disappointed, but the thing

Mato Inc.Notes to the Financial Statements as at February 28, 2007

1. Significant accounting policies:a. Inventory is valued at the lower of cost, determined by the first-in, first-out

method, and net realizable value.b. Noncurrent assets are recorded at cost. Amortization is calculated on a straight-

line basis of 20% of cost per annum on automotive equipment, leaseholdimprovements, and computer equipment and software, and at 10% of cost perannum on other equipment and furniture.

2. The bank loan is secured by receivables, inventories, a general charge on thecompany’s assets, and by the personal guarantees of the shareholders.

3. The company’s authorized capital is 1,000,000 shares without par value. At thebeginning of the year, 12,500 shares were issued for $10 cash each.

4. No provision for income taxes has been made in the financial statements becausethe current loss will result in an income tax recovery only if there are future taxableincomes against which that loss may be deducted.

5. Salaries of directors and officers of the company were $54,280 for the year.6. The company has commitments to purchase goods that will cost $23,430 on

delivery, which is expected by April 30, 2007.

FinancialAcct-C09-v3 3/28/06 2:56 PM Page 610

NEL 611

to do is to try harder to manage the company well, not “shoot the messenger” bytrying to change the financial accounting “message.”

2. Such a change would not likely help in dealing with the parties who are going to bemost concerned about the company. The bank already has the company’s assetspretty well tied up as security on its loan and is going to be interested in thecompany’s ability to generate cash to repay the loan, as well as in its long-term viabil-ity. The bank is undoubtedly very concerned about the company and will be on thelookout for desperate actions, so optimistic reporting is unlikely to fool thebankers—or, for that matter, suppliers, other investors, and the company’semployee.

3. The cash flow statement would show the same cash flow figures regardless ofaccounting policy changes, so users of the financial statements who know how toread the cash flow statement would see through such changes, and might evenbecome suspicious if the income diverged too much from the cash flow from operations.

4. For this company, there is really not much that could be manipulated even if it wereethical and successful. Receivables are not large, nor is there any obvious reason forthe company to recognize revenue sooner than it does without violating GAAP.Inventories are also not large, and, since FIFO is already being used, there is likelyto be little room for raising inventory value to increase income. Amortization couldbe slowed down, but such a move would make little difference to income. Evencutting amortization in half would reduce the year’s loss by less than 20%.

FinancialAcct-C09-v3 3/28/06 2:56 PM Page 611

Homework and Discussion to Develop Understanding

The problems roughly follow the outline of the chapter. Three main categories of questions are presented.

• Asterisked problems (*) have an informal solution provided in the Student SolutionsManual.

• EXTENDED TIME problems grant a thorough examination of the material andmay take longer to complete.

• CHALLENGING problems are more difficult.

For further explanation, please refer to Section 1.15.

Current and Noncurrent Liabilities • Section 9.2 and 9.3

Understanding Liabilities

* PROBLEM 9.1 Questions about the right-hand side of the balance sheet

Answer the following questions briefly:

1. What is the difference between liabilities and equity?2. What is the difference, if any, between liabilities and debts?3. Suggest two examples each of short-term and long-term accruals that require

difficult estimates and indicate what the difficulty is in each case.4. Should companies avoid long-term accruals because they are likely to be inaccurate

and therefore misleading, and just pay in cash costs such as warranties and pensionsas they arise?

PROBLEM 9.2 Answer questions about liabilities

Respond to the question asked in each comment below.

1. “Warranties are honoured as part of good business practice: keep your customershappy. Whether to honour a claim and how much cost to incur are managerial judg-ments that depend on how good the customer is, what the reputation effects are,etc. Therefore, warranties are discretionary period expenses like donations. There isno accrual for future donations, so why is there an accrual for future warrantycosts?”

2. “Employees take their pensions when they retire, years into the future. Whetherthere is any pension depends on whether the employee keeps working for thecompany, and how long the employee lives after retirement. Therefore pensioncosts can only be realistically determined when they are being paid in the future.Why shouldn’t they be expensed then, rather than now?”

3. “After thinking about warranties and pensions, I have a proposal. Let’s take accrualaccounting to the next step and accrue all future expenses we can reasonablypredict now. I mentioned donations already. We could add repairs and mainte-nance, interest, income taxes, and executive bonuses, to mention just a few.”

PROBLEM 9.3 Effects of pension accounting on the future

Here’s a problem. Suppose companies in general did not accrue enough pension liabili-ties to account fully for their obligations to employees, warranty liabilities to account fortheir obligations to customers, etc. There are persistent worries about this in these timesof aging baby boomers and less complacent consumers. A commentator said, “The realcosts of company obligations are not being recognized in balance sheets and so cash isbeing taken out of the hands of future generations of employees and consumers.”

9.10

612 PART THREE DOING FINANCIAL ACCOUNTING

NEL

hallenging andC

E xtended time

FinancialAcct-C09-v3 3/28/06 2:56 PM Page 612

Explain how the alleged failure to accrue enough for obligations like pensions andwarranties could take cash out of the hands of future generations.

Liability Calculations

* PROBLEM 9.4 Do current and noncurrent liability calculations

In each case below, calculate (1) current liability at the end of this year; (2) noncurrentliability at the end of this year; and (3) interest expense for the next year.

a. John Ltd.’s factory mortgage of $842,500 requires payments of $11,200 each month.During the next year, the interest part of the payments will equal $61,232.

b. Frieda Inc.’s land mortgage of $232,200 requires payments of $60,000 over the nextyear. By the end of next year, the principal due on the mortgage will have gonedown to $189,400.

c. Graham Ltd.’s $87,436 property mortgage requires monthly payments of $1,500plus interest. During the next year, payments will total $25,674.

PROBLEM 9.5 Questions about long-term accruals

1. Balmer Inc. started the year with a pension liability of $42,000. During the year,employees earned pension entitlements with a present value of $147,600 and thecompany paid $157,400 to the pension trustee. State the amount of the pensionexpense for the year and the pension liability at the end of the year.

2. Balmer Inc. also has a warranty plan. Estimated warranty liability was $64,000 at thebeginning of the year, and based on the company’s sales for the year, warranty serv-ice costing $100,500 in wages and other costs, plus $79,400 in replacement prod-ucts, was expected to have to be provided eventually. Actual expenditures for theyear were $107,500 in wages and other costs and $99,800 in replacement products.Calculate warranty expense for the year and estimated warranty liability at the endof the year.

3. Write one or more journal entries to record Balmer Inc.’s warranty experience forthe year.

4. For part 2, what would be your answers if Balmer Inc. accounted for warranties onthe cash basis instead of the accrual basis?

5. Suppose Balmer Inc. had a real disaster with a product during the next year. Forthat product only, wages and other service costs totalled $150,000 and replacementproducts costing $210,000 had to be provided. Comment on what this disastermight do to the company’s warranty accounting. Focus on the accounting issues—no numbers are needed.

PROBLEM 9.6 Calculate current and noncurrent portions of a mortgage

Tweedsmuir Land Ltd. has substantial mortgage debt. The debt was $15,254,182 at thebeginning of this year. Mortgage payments of $4,394,051 were made during the year andthe mortgage balance was $12,261,968 at the end of the year. No new borrowing wasmade this year. Next year, the payments required total $4,639,373 and if there is no newborrowing the mortgage balance will be $8,749,453 at the end of next year. Calculate (a) Interest expense for this year; (b) Current portion of the mortgage liability at theend of this year; and (c) Noncurrent mortgage liability at the end of this year.

PROBLEM 9.7 Calculate various liability and expense amounts

In each of the following cases, calculate the year-end liability and any associated expensefor the year.

CHAPTER 9 LIABILITIES, EQUITY, AND CORPORATE GROUPS 613

NEL

FinancialAcct-C09-v3 3/28/06 2:56 PM Page 613

1. Bach Inc. has collected fees from orchestras for their performance of varioussymphonies to which it has the rights. During the year, it collected fees of $250,000,but by the end of the year, the orchestras had performed symphonies correspon-ding to only $192,000 of the fees. The rest were to be performed next year.

2. Beethoven Ltd. started the year owing $138,000 to its pension plan trustee. Duringthe year, its employees earned pensions having an estimated present value of$439,500, and the company paid the trustee $489,900.

3. Redo part 2 changing only one number: change the present value of pensionsearned to $333,200. Where would the liability figure you got appear on the balancesheet?

4. Mozart Inc. has a warranty plan for the musical instruments it manufactures. Thecompany’s prices are high and its quality is good, but musicians are a choosy lot, sothe company has a substantial warranty cost, which its management feels is worth-while to keep its reputation and repeat business up. At the beginning of the year,the estimated warranty liability was $218,320. During the year, the companyincurred warranty costs of $142,280, $97,640 of that in cash and the rest in replace-ment instruments. This was close to the $140,000 expected on the basis of the year’ssales.

5. Suppose Mozart Inc. had an unexpected problem with a new model of tuba andrecalled all the tubas sold for free repairs, costing the company $75,000 in total.Knowing this, would this change either of your answers to part 4?

PROBLEM 9.8 Calculate various liability and expense amounts

Calculate the unknown figure in each of the following unrelated cases.

1. Current portion of long-term debt = $983,500. Payments due next year = $2,480,000.Total debt = $8,554,000. How much interest is to be paid next year?

2. Net bonded debt liability on the balance sheet = $22,380,720. Interest to be paidnext year = $1,849,750. Discount amortization will be $67,000. What will the interestexpense be for next year? What will be the net bonded debt liability on the balancesheet at the end of next year?

3. Pension liability at the beginning of the year = $899,900. Payments to the pensiontrustee during the year = $3,679,280. Liability at the end of the year = $323,610.What was the estimated present value of the pensions earned during the year? Whatwas the pension expense for the year?

4. Warranty costs are accounted for on the cash basis, and this year the warrantyexpense = $182,920. An analysis of the sales and warranty patterns indicates that esti-mated warranty obligations were $62,040 at the beginning of the year and $106,490at the end. If accrual accounting were used for warranties, what would the warrantyexpense be for this year? What difference would it make to income before incometax?

Bond Discounts and Premiums

* PROBLEM 9.9 Bond discount or premium calculations

In each case below, (1) calculate the amount of any discount or premium on issue of thebonds, (2) record the issue of the bonds, and (3) state whether interest expense overthe life of the bonds will be higher, lower, or the same as the cash interest paid on thebond each year.

a. A Ltd. issued 10,000 $100 bonds and received $97.50 cash each.b. B Ltd. issued 10,000 $100 bonds, but found that the planned interest rate of 7% was

lower than market rates and so received $915,000 for them.c. C Ltd. issued 10,000 $100 bonds for a premium of 5% on legal value.

614 PART THREE DOING FINANCIAL ACCOUNTING

NEL

FinancialAcct-C09-v3 3/28/06 2:56 PM Page 614

PROBLEM 9.10 Record a new bond issue

Sunshine Vacations Inc. (SVI) is a large tour operator in North America. In order tofinance a new expansion into the Caribbean, the company hopes to raise money byoffering an 8.5% bond issue worth $42,000,000. The bonds were well received by themarket and ended up raising $46,200,000 yielding the current market rate.

a. Record the bond issue.b. When the bond premium is amortized, will it increase or decrease the overall inter-

est expense on the bonds? (Hint: will the overall interest expense be debited orcredited?)

c. Does this increase or decrease in interest expense mean that the company is payingan effective interest rate higher or lower than the coupon rate of 8.5%?

PROBLEM 9.11 Calculate or record various liability and expense amounts

1. High Risk Inc. issued $10,000 first-mortgage bonds having a total face value of$50,000,000 and carrying a 6.2% interest rate. They were not well received by thebond market, so High Risk received only $47,074,275 for them. (a) Record the bondissue, and (b) calculate the balance sheet liability for the bonds as of the issue date.

2. Regarding part 1, will High Risk’s interest expense for the bonds be more or lessthan 6.2% times the face value ($3,100,000)? Why?

PROBLEM 9.12 Answer questions about bond discounts and premiums

The president of Redstone Inc. has just been discussing a planned new bond issue withfinancial advisors and has come to you with some resulting accounting questions.Answer each.

1. “We are thinking of setting the interest rate on the bonds a little above currentmarket rates to make them more attractive. I’m told this will produce a balancesheet liability higher than the face value of the bonds. How does this happen?”

2. “Your answer to my first question is troubling. How can it make sense to show aliability higher than the amount we will have to pay when the bonds come due?What will we do with the difference then?”

3. “So I guess the amount of interest expense on the bonds, as calculated by theaccountants, will be less than the actual amount of interest paid each year. That willbe nice, because the reduction will improve earnings and carry through to our cashflow statement, right?”

Explaining Income Tax Accounting

* PROBLEM 9.13 Explain some things about income tax allocation

Explain the following:

1. Why is it thought necessary to have a future portion of income tax expense andfuture income tax liability?

2. How does the resulting income tax allocation method fit accrual accounting?3. Why is the calculation method of the future portion of the income tax expense

sometimes described as not matching revenue very well?

PROBLEM 9.14 Temporary differences in income tax accounting

Explain what “temporary differences” are, and why the future income tax liability (orasset) depends on them but not on permanent differences between accounting andincome tax calculations such as nontaxable revenues or nondeductible expenses.

CHAPTER 9 LIABILITIES, EQUITY, AND CORPORATE GROUPS 615

NEL

FinancialAcct-C09-v3 3/28/06 2:56 PM Page 615

PROBLEM 9.15 Explain the purpose and nature of income tax allocation

George picked up the financial statements of a company he owns shares in and noticedthe following two accounts, which he didn’t understand:

Future portion of income tax expense $19,749,200Future income tax liability $86,293,500

Explain to George the purpose of income tax allocation accounting and what the twofigures he didn’t understand mean.

PROBLEM 9.16 Answer questions about income tax accounts

Answer each of the following questions:

1. Before income tax allocation accounting was introduced, the income tax expenseon companies’ income statements just equalled the income tax payable on eachyear’s taxable income. For a company with new factory assets, would net incomehave been larger or smaller without income tax allocation? Explain.

2. Following from part 1, would net income have been larger or smaller withoutincome tax allocation for a company with old factory assets? Explain.

3. Companies do not have to claim capital cost allowance deductions in calculatingtheir taxable income if they choose not to. Thus, there are circumstances in whichthe full CCA available is not claimed. If a company chooses not to claim the fullCCA available, what will that do to (i) current portion of income tax expense, (ii) future portion of income tax expense, (iii) total income tax expense?

4. What are the effects of each of the following on (i) current income tax expense, (ii) future income tax expense, (iii) total income tax expense?a. A company incurs and pays a large repair expense.b. A company buys a large new machine for its factory.c. A company buys a large shipment of inventory to be sold next year.d. A company receives a large nontaxable dividend from another company.e. A company declares and pays a dividend to its shareholders.f. A company makes a large sale to a good customer.

Calculating Income Tax

* PROBLEM 9.17 Calculate income tax expense and future tax liability

At the end of 2005, Henrik Inc. had future income tax liability of $329,612 and retainedearnings of $3,949,286. For 2006, the company’s income statement showed incomebefore tax of $648,960. There was only one temporary difference between the account-ing and income tax calculations for 2006: capital cost allowance (CCA) exceeded bookamortization expense by $343,502. There was also $29,650 of nontaxable revenue, so thecompany’s taxable income was $275,808 for 2006. The company’s income tax rate for2006 was 32% and is expected to remain at that rate indefinitely. The company paid nodividends in 2006.

1. Calculate the following:a. Current portion of income tax expense for 2006.b. Future portion of income tax expense for 2006.c. Net income for 2006.d. Future income tax liability at the end of 2006.e. Retained earnings at the end of 2006.

2. Suppose the tax law changed a little in 2006, so that expected income tax rates inthe future were now expected to vary from 32%, and the estimated future income

616 PART THREE DOING FINANCIAL ACCOUNTING

NEL

E xtended time

FinancialAcct-C09-v3 3/28/06 2:56 PM Page 616

tax liability at the end of 2006 is now $420,500. Recalculate the five numbers askedfor in part 1.

* PROBLEM 9.18 Calculate net income and future income tax liability

Mars Bears Ltd. has a stable income before income tax on its income statement:$120,000 each year. Its income tax rate is also stable: 36%. It has one asset costing$350,000 that it amortizes at $35,000 per year. The capital cost allowance deductible onthat asset is 10% in the first year and 20% in each year thereafter, the rate being appliedto the unamortized cost (CCA is a declining balance method). Calculate the followingfor each of the first three years of the company’s existence:

a. CCA deductible.b. Taxable income (add book amortization back to income and deduct CCA instead).c. Current portion of income tax expense.d. Future portion of income tax expense.e. Total income tax expense.f. Net income for the year.g. Future income tax liability at the end of the year.

PROBLEM 9.19 Calculate net income, future income tax liability, and other amounts

You have the following information about Gazoo Entertainments Ltd.:

Current Future Future Income

Income Before Income Tax Income Tax Tax Liability at

Year Income tax Expense Expense Beginning of Year

2005 $409,125 $101,250 $51,150 $142,530

2006 538,340 129,100 45,450 ?

2007 217,350 47,300 27,610 ?

2008 (51,100) (27,800) 9,250 ?

2009 42,775 27,480 (5,270) ?

Answer the following:

a. What is the net income in 2006, 2008, and 2009?b. What is the future income tax liability at the end of 2009?c. What is the total income tax expense for 2008?d. In 2008, how could there be both a negative current income tax expense and

positive future income tax expense?e. Assuming Gazoo’s future income tax arises from differences between book

amortization and CCA, is amortization greater than CCA in 2005? In 2007? In 2009?f. What was the company’s effective income tax rate in 2007? In 2008?

PROBLEM 9.20 Calculate income tax expense and future tax liability

At the end of 2006, Plasticorp Ltd. had future income tax liability of $2,513,140 andretained earnings of $25,492,725. For 2007, the company’s income statement showedincome before tax of $2,211,305 and amortization expense of $4,382,436. Inspection ofthe company’s income tax records showed that in 2007, $88,860 of its revenue was notsubject to income tax, $50,557 of its expenses were not tax deductible, and its capitalcost allowance was $4,045,730, so its taxable income was $2,1509,706. The company’sincome tax rate for 2007 was 35% and is expected to remain at that rate indefinitely.The company declared and paid a $500,000 dividend in 2007.

CHAPTER 9 LIABILITIES, EQUITY, AND CORPORATE GROUPS 617

NEL

E xtended time

FinancialAcct-C09-v3 3/28/06 2:56 PM Page 617

1. Calculate the following:a. Current portion of income tax expense for 2007.b. Future portion of income tax expense for 2007 (the difference between book

amortization and CCA was the only temporary difference in 2007).c. Net income for 2007.d. Future income tax liability at the end of 2007.e. Retained earnings at the end of 2007.

2. Suppose expectations of future income taxes to be paid changed a little in 2007, sothat the estimated future income tax liability at the end of 2007 is now $2,340,500.Recalculate the five numbers asked for in part 1.

PROBLEM 9.21 Journal entry for income tax payable and future tax

For 2006, Great World Air Inc. had income before income tax of $26,600 (in thousandsof dollars). Taxable income for 2006 was $23,760 thousand, and an analysis of balancesheet accounts indicated that temporary differences between accounting and tax rulesof $3,737 thousand arose in 2006. The company’s income tax rate for 2006 was 36% andthat is expected to be the rate for the future.

1. Write a journal entry to record the company’s 2006 income tax expense. Show yourcalculations.

2 Based on your answer to part 1, what was the company’s net income for 2006?

Equity • Section 9.4

Understanding Equity Accounting

* PROBLEM 9.22 Explain some features of accounting for equity

Answer each of the following questions briefly:

1. Why does an unincorporated business have only a single equity account, Capital?2. What does it matter that a bond or preferred share may be convertible to a common

share?3. Why is a stock split ignored in accounting whereas a stock dividend is traditionally

recorded (debit retained earnings, credit share capital)?4. Why is there so much disclosure of the legal details of shares and other equity

accounts?5. Why does it matter if a class of shares has a minimum issue price or par value?

PROBLEM 9.23 Answer various questions about equity accounting

Answer each of the following questions briefly.

1. Give some examples of distinctions made in financial accounting between equityaccounts and other balance sheet accounts that are in some way problematic. Foreach, what is the problem?

2. Given the increasing number of hybrid and generally complex financial instrumentsand financing methods, should financial accounting drop the debt-equity distinction on the right-hand side of the balance sheet?

3. The business’s economic earning power, assets, and liabilities are presumably whatmatters to external users of financial accounting information. Why then are therevarious rules about accounting for equity that depend on the form of the businessorganization (e.g., whether it is incorporated or not)?

4. The equity section of the balance sheet is recorded at historical amounts, not at the current market value of the equity, what the company could be sold for today.

618 PART THREE DOING FINANCIAL ACCOUNTING

NEL

FinancialAcct-C09-v3 3/28/06 2:56 PM Page 618

Given that, what aspects, if any, of the way equity is accounted for provide usefulinformation to the owners of the equity?

PROBLEM 9.24 Calculations of partnership equity

Big Partners is a firm of architects, whose partners are Barbara, Ian, and Gordon. Thepartnership agreement specifies that the first two partners get 30% each of the firm’sincome and the third partner gets 40%. During the most recent year, the firm earned$600,000 before any withdrawals by partners. At the end of the year, the firm’s liabilitiestotalled $435,400. At the beginning of the year, Barbara’s capital account was $105,000;Ian’s was $23,900; and Gordon’s was $183,200. Withdrawals for the year were: Barbara,$143,600; Ian, $150,100; and Gordon, $61,000.

1. Calculate the partners’ capital accounts as at the end of the year.2. Present the partnership’s summarized balance sheet as at the end of the year.3. Suppose Ian had withdrawn $225,000 instead during the most recent year. Would

you think the partnership’s balance sheet would or should be different from the oneyou presented in part 2? Why, or why not? This has not been covered in the chapter,but you should be able to consider the question on the basis of your knowledge of asset and equity accounting.

Financial Instruments • Section 9.5

Disclosure of Financial Instruments

PROBLEM 9.25 Investment classification

Ergometric Ltd., whose shares are broadly traded on the TSX, has numerous invest-ments. As the newest accounting clerk, you have been assigned to classify the followinginvestments and recommend the method to be used in accounting for each of them.

a. Various long-term government of Canada bonds maturing in ten to twenty years areacquired as a temporary investment. It is management’s expectation that interestrates will fall, causing a significant price increase in the bonds within the next year.

b. An investment in bonds is held as part of an actively traded portfolio. The moneyinvested in the bonds is obtained from loans and Ergometric is cautious in matchingthe maturities of the loans and the bonds acquired.

c. Ergometric holds 8%, the largest single shareholding, of the voting shares of a largepublic company. Ergometric is proportionately represented on the board of direc-tors. Management intends to hold the shares for a long time, awaiting significantappreciation in the market price.

d. Ergometric owns 45% of the voting shares of a large public company. The remain-ing 55% of shares are widely held. Ergometric elected 4 of the 10 members of theboard of directors and generally sets the operating and financing policy of the firm.

e. Ergometric holds a tiny percentage of the shares in a large, public company. Thevalue of these shares has increased dramatically but they have not been sold.Management is convinced the shares will appreciate even more and as a resultborrowed money, using the share as collateral, when cash was needed.

PROBLEM 9.26 Investment classification

The shares of Wise Inc., a manufacturer of owl figurines, are traded on the TSX. Wisehas been very profitable in recent years. Rather than diversify or distribute the earningsas dividends, Wise has made a number of investments. As the new accountant, you areasked to classify the investments listed below and indicate the recommended method ofaccounting for each.

CHAPTER 9 LIABILITIES, EQUITY, AND CORPORATE GROUPS 619

NEL

FinancialAcct-C09-v3 3/28/06 2:56 PM Page 619

a. Wise holds $1,500,000, 6% bonds that mature in 25 years. Wise will sell the bonds ifthe market price rises sufficiently or if the money is needed for expansion.

b. Wise holds $5,000,000 in bonds of Seagull Ltd. The bonds mature in 2020 and,because of the high interest yield, management expects to retain the investmentuntil then.

c. Wise holds 10,000 shares of the Royal Bank. These shares are held in anticipation ofcapital appreciation.

d. Wise holds 60% of the common shares of Garden Gnome Inc.; the remaining 40%are held by Gary Gardener. The board of directors of Garden Gnome consists of sixmembers, four of whom are appointed by Wise Inc. All decisions of the board mustbe unanimously agreed to by the board members.

e. Wise holds 30% of the shares of Flightless Inc. The remaining shares are equallydivided among three other investors. Each investor has equal representation on theboard of directors. The other representatives are quite passive and attend meetingsirregularly. All strategic decisions to date appear to have been made by the Wiseappointees.

PROBLEM 9.27 Effects of pension assets on the financial statements

Respond to the following events at Millwood Corp.:

Millwood Corp. has a pension plan covering substantially all its employees. The pensiontrustee is a separate legal entity from Millwood, but a majority of its directors are alsosenior managers at Millwood. At the end of the year, the pension trustee calculated thatits expected obligation for employees’ pensions (present value of expected payoutsdiscounted at projected rates of return on pension assets) was $778 million. At the samedate, the trustee had assets of $814 million. The only pension account on Millwood’sbalance sheet at that date was a liability to the trustee of $6 million, representingpension deductions and employee withholdings from the latter part of the year. (The $6million was shown as a receivable in the trustee’s balance sheet; so was part of its $814million of assets.)

1. In a speech, the CEO of Millwood referred to an “off-balance-sheet asset” of “over$800 million.” A critic of the speech said the number was only $36 million. Do youthink it was either? Explain.

2. In a management meeting, the Millwood CEO mentioned that the company shouldplan to “recover” the trustee’s excess assets of $36 million and take that amount intoincome. What do you think about that plan?

3. Suppose all the Millwood data were the same except that the trustee’s estimatedobligation was $852 million at the end of the year? Does this change either of youranswers above?

4. What about the liability side of the Millwood case? Using the estimated obligation of$852 million, what adjustment (if any) to the liabilities side of Millwood’s balancesheet would you make? What disclosure would you make in Millwood’s notes? Arethere any implications for Millwood’s income?

PROBLEM 9.28 Adequacy of GAAP for financial instruments

This chapter has had several topics in which controversy exists over the way GAAP do ordo not deal with modern financial activities. There is the problem of financing that isnot clearly either debt or equity. There is the attempt to make estimates of futureincome tax payments, which may be made only in the distant future. A major ongoingproblem is in accounting for financial instruments. Or perhaps it would be better to say“lack of accounting for financial instruments” because while GAAP require considerabledisclosure about such instruments, there is little progress on accounting for them bybuilding them, and associated gains and losses, into the financial accounting numbers.

620 PART THREE DOING FINANCIAL ACCOUNTING

NEL

E xtended time

hallengingC

FinancialAcct-C09-v3 3/28/06 2:56 PM Page 620

Is disclosure enough? Should “mark to market” accounting be required, so that theaccounts reflect the kinds of calculations that financial managers make when makinghedging, interest rate swap, and other financial arrangements?

Discuss the question of whether financial accounting is now too far removed fromthe way modern financial arrangements are made, and if so, what might be done aboutit. Would it be reasonable to change the accounting every time an innovative financialinstrument was developed? Or is the problem deeper, involving a fundamental problemwith financial accounting’s historical cost base that is most apparent when financialinstruments and financial arrangements are, in general, considered?

Corporate Groups: Intercorporate Investment • Section 9.6

Cost Versus Equity Investment Calculations

* PROBLEM 9.29 Cost versus equity basis for nonconsolidated investment

China Sports Ltd. purchased 40% of the voting shares of Brassy Ltd. at the beginning ofthis year for $4,100,000. During the year, Brassy earned net income of $600,000 and paiddividends of $250,000. China Sports, which has been accounting for its investment inBrassy on the cost basis, has income of $800,000 for this year. If the equity basis wereused instead, what would China Sports Ltd.’s income be?

PROBLEM 9.30 Cost versus equity basis of an investment

Widgets-for-u Inc. increased its investment in Widgetland Inc. from 15% to 25% onJanuary 1, 2006, the first day of the fiscal year. As a result, Widgets-for-u Inc. has changedits accounting policy to account for Widgetland Inc. on the equity basis instead of on acost basis. For the year ended Dec. 31, 2005, Widgetland Inc. had net income of$250,000 and paid dividends of $85,000. For the year ended Dec. 31, 2006, WidgetlandInc. increased its net income to $550,000 and declared dividends of $210,000. Whatfigure did Widgets-for-u Inc. report in its income statement as revenue from its investment in Widgetland Inc. in 2005 and 2006?

PROBLEM 9.31 Equity method

On January 1, 2006, Harrison Co. purchased 40% of the outstanding common (voting)shared of Junior Corporation for $40,000. This gave Harrison significant influence. Atthe date of acquisition, the balance sheet of Junior showed the following book values:

Book Value Market Value

Assets not subject to amortization $50,000 $50,000

Assets subject to amortization 35,000 40,000

Liabilities 10,000 10,000

Common shares 65,000 N/A

Retained earnings 10,000 N/A

1. Prepare the journal entry to record the acquisition.2. Show your calculation of purchased goodwill at the date of acquisition.3. Junior reported net income for the year ended December 31, 2006, in the amount

of $14,000 and paid dividends of $4,500 during the year. The assets subject to amor-tization have a remaining useful life of five years and straight-line depreciation isused. Goodwill has not been impaired. Prepare the entries Harrison Co. wouldprepare.

CHAPTER 9 LIABILITIES, EQUITY, AND CORPORATE GROUPS 621

NEL

FinancialAcct-C09-v3 3/28/06 2:56 PM Page 621

Corporate Groups: Consolidation • Section 9.7

Understanding Consolidation

PROBLEM 9.32 Explanation of consolidated statements

In 2006, Parent Company acquired 80% of the outstanding voting shares of SubsidiaryCompany, establishing control over the board of directors. Parent Company used thecost method of accounting for the investment during the year, but prepared consoli-dated financial statements at the end of the year. The consolidated financial statementsare summarized below:

Parent Subsidiary Consolidated

Cash $ 14,000 $ 15,000 $ 29,000

Accounts receivable 27,000 24,000 45,000

Inventory 18,500 11,500 30,000

Property, plant and equipment 110,000 80,500 190,500

Investments in Subsidiary Company 92,000

Intangible assets 5200

$261,500 $131,000 $299,700

Current liabilities $ 37,000 $ 11,500 $ 42,500

Long-term liabilities 5,000 3,000 8,000

Minority interest 25,200

Common shares 125,000 75,000 125,000

Retained earnings 94,500 41,500 99,000

$261,500 $131,000 $299,700

Revenue $110,000 $ 77,000 $167,000

Cost of goods sold 70,000 42,500 96,500

Operating expenses 22,000 21,500 43,500

Minority interest 4,500

Net income $ 18,000 $ 13,000 $ 22,500

Retained earnings, beginning of year $ 89,000 $ 33,500 $ 89,000

Dividends 12,500 5,000 12,500

Retained earnings, end of year $ 94,500 $ 41,500 $ 99,000

Required:

1. Why does Parent Company use the cost method throughout the year? Could Parentuse an alternative method?

2. Explain the meaning of the accounts appearing on the consolidated balance sheetthat do not appear on either of the unconsolidated balance sheets.

3. Certain accounts and amounts from the unconsolidated statement do not appearon the consolidated statements. Identify these amounts and accounts and explainwhy they are eliminated in the consolidation.

4. Accounts receivable and accounts payable on the unconsolidated statement do nottotal to the amount shown on the consolidated statements. What is the most likelyreason for this?

622 PART THREE DOING FINANCIAL ACCOUNTING

NEL

FinancialAcct-C09-v3 3/28/06 2:56 PM Page 622

PROBLEM 9.33 Answer conceptual questions on consolidation

Chromium Furniture Ltd. wishes to expand operations by acquiring other furnituremanufacturers and associated businesses. In relation to this, the president is curiousabout accounting methods for groups of companies. Answer briefly the following fourquestions the president has asked:

1. “Why does a subsidiary have to be consolidated with the parent’s accounts?”2. “Why doesn’t consolidating a newly acquired subsidiary affect consolidated retained

earnings? (After all, the subsidiary has retained earnings too.)”3. “Since it is the sum of more than one company, won’t a consolidated balance sheet

present a stronger financial picture than the parent’s unconsolidated balance sheetdoes?”

4. “What does ‘Goodwill on consolidation’ on the consolidated balance sheet mean?”

PROBLEM 9.34 Problems with goodwill on consolidation

In 2001 and 2002, there were many very large reported losses related to goodwill write-offs. JDS Uniphase and AOL Time Warner, for example, both reported losses in theU.S.$50 billion range. Both companies had recorded the goodwill from acquisitions(JDS of Uniphase, AOL of Time Warner) that had been paid for by the issuance ofshares of the acquiring company, valued at the huge stock market values typical of high-flying tech companies in the late 1990s.

1. Explain how the large goodwill values in such companies’ consolidated balancesheets arose. Which other figure on the balance sheet was also made very large bythe acquisition?

2. What does the answer to part 1 do to the usefulness of standard ratios like debt-equity and working capital for evaluating such companies’ financial position?

3. When the value of tech companies collapsed in the early 2000s, what should havebeen done with the apparently inflated values of (a) consolidated goodwill and (b) share capital? Did AOL and JDS really have any option but to report the staggeringlosses they did?

Consolidation Calculations

* PROBLEM 9.35 Basic consolidation calculations and balance sheet

Seeking to expand its markets, Big Ltd. recently purchased 80% of the voting shares ofPiddling Ltd. for $10,800,000. At the date of the acquisition, Piddling had assets of$14,600,000, liabilities of $8,200,000, and equity of $6,400,000. By the best estimate Bigcould make at the date of acquisition, the fair market value of Piddling’s assets was$16,100,000 and that of its liabilities was $8,300,000.

1. Calculate the goodwill on consolidation as of the acquisition date.2. Calculate the noncontrolling (minority) interest as of the acquisition date.3. Complete the consolidated balance sheet figures below.

Account Big Ltd. Piddling Ltd. Consolidated

General assets $105,000,000 $14,600,000 $Investment in Piddling 10,800,000GoodwillGeneral liabilities 83,700,000 8,200,000Noncontrolling interestEquity 32,100,000 6,400,000

CHAPTER 9 LIABILITIES, EQUITY, AND CORPORATE GROUPS 623

NEL

FinancialAcct-C09-v3 3/28/06 2:56 PM Page 623

PROBLEM 9.36 Basic consolidated figures

Fine Furniture has decided to purchase 65% of Steel Appliances Ltd. for $38,000,000 incash. The two companies’ balance sheets as at the acquisition date are (in millions ofdollars):

Assets Liabilities and Equity

Fine Steel Fine Steel

Cash equivalent assets $112 $10 Cash equivalent liabilities $ 28 $ 0Other current assets 304 45 Other current liabilities 260 10Noncurrent assets (net) 432 25 Noncurrent liabilities 272 15

Share capital 160 15Ret. earnings 128 40

$848 $80 $848 $80

Fine Furniture has evaluated all of Steel’s assets and liabilities as having fair value equalto book value except for its noncurrent assets, which Fine Furniture believes have a fairvalue of $28 million.

1. Calculate the consolidated goodwill that would appear on the consolidated balancesheet at the acquisition date.

2. Calculate the following consolidated figures as at the acquisition date:a. Consolidated total assets.b. Consolidated owners’ equity.c. Consolidated total liabilities.

PROBLEM 9.37 Goodwill amount and reasons; later consolidated income

Trifecta Ltd. recently purchased a 70% interest in Mega Magazines, a small magazinewholesaler. Mega’s balance sheet on the date of acquisition appears below.

Assets Liabilities and Equity

Cash $ 10,000 Liabilities $ 95,000Accounts receivable (net) 58,000 Owners’ equity 108,000Inventory 60,000Fixed assets (net) 75,000

$203,000 $203,000

Mega’s receivables have an adequate provision for doubtful accounts. Inventories arecarried at cost and current replacement value is about $60,000. Land with a book valueof $30,000 has a market value of $39,000. In the purchase agreement, Trifecta assumedall of Mega’s liabilities. Before the sale was final, the then owners of Mega were allowedto withdraw all cash from the company as a dividend.

1. If Trifecta paid $110,000 (in addition to the $95,000 to pay the liabilities) for itsinterest in Mega Magazines, what was the amount of purchased goodwill? (Hint: allTrifecta got for its money were receivables, inventories, and fixed assets.)

2. Why would Trifecta have been willing to pay this amount for goodwill?3. Assume that in the year following the acquisition, Mega made a net income of

$24,000. Therefore, decide whether the following statement is true or false and statewhy: To record Mega’s earnings, the consolidated retained earnings of Trifecta willbe increased by $24,000.

624 PART THREE DOING FINANCIAL ACCOUNTING

NEL

FinancialAcct-C09-v3 3/28/06 2:56 PM Page 624

CHAPTER 9 LIABILITIES, EQUITY, AND CORPORATE GROUPS 625

NEL

PROBLEM 9.38 Equity basis of accounting versus consolidation

Accounting for intercorporate investments is subject to GAAP. Apply your knowledge ofGAAP to the following situation.

Newsworthy Inc. (NI) owns 45% of the voting shares of Reportit Corp. (RC). NIacquired the shares several years ago for $8,500,000. RC lost money for a few years afteracquisition but has recently begun to be profitable: since NI acquired its shares, RC hashad losses totalling $750,000 and incomes totalling $1,025,000, for a total net incomesince acquisition of $275,000. Last year, RC paid its first dividend, $50,000.

1. NI accounts for its investment in RC on the equity basis. What does this mean?2. What is the present figure for investment in RC on the balance sheet of NI?3. What difference would it make to the balance sheet of NI if the RC investment were

consolidated instead?4. Suppose that NI had bought 65% of the RC voting shares for its $8,500,000 and that

at that date the following values existed for RC: book value of assets, $15,000,000;sum of fair values of assets, $16,000,000; book value of liabilities, $5,000,000; sum offair values of liabilities, $8,000,000. Calculate the goodwill that would have beenshown on the consolidated balance sheet of NI if the RC investment had beenconsolidated at that date.

* PROBLEM 9.39 Accounting effects of a business acquisition

Suppose that, to spread its business risk, a major brewing company decides to buy intothe retail furniture business by acquiring a controlling interest in a national chain thatsells furniture, appliances, and related goods at discount prices. Changing its usualpolicy of 100% ownership, the brewer acquires a 60% voting interest. On January 1,2005, the brewer pays $54,000,000 cash for 60% of the chain’s voting shares. At thatdate, the chain’s balance sheet shows:

Cash $ 2,000,000 Demand bank loan $14,000,000Other current assets 53,000,000 Other current liab. 26,000,000Noncurrent assets 38,000,000 Noncurrent liab. 20,000,000Less accum. amort. (6,000,000) Shareholders’ equity 27,000,000

$87,000,000 $87,000,000

The brewer’s evaluation is that the fair values at January 1, 2005, are the same as thebook value for all of the chain’s assets and liabilities except land; it is on the chain’sbooks at a cost of $4,000,000, but the brewer’s evaluation is that its fair value is$7,000,000 at January 1, 2005.

The furniture chain is expected to report a substantial net income for the fourmonths from January 1, 2005, to April 30, 2005 (the brewer’s year-end), so the brewerymanagers are pleased with their decision to get into the furniture retailing business.

1. The brewer owns more than 50% of the furniture chain, so it should be consoli-dated with the brewer’s other companies in preparing financial statements. But thechain is quite different from the brewer’s other activities, so would it make sense toadd apples in with oranges? Comment on this question.

2. Calculate the consolidated goodwill (if any) arising as at January 1, 2005, from thepurchase of the furniture chain.

hallengingC

FinancialAcct-C09-v3 3/28/06 2:56 PM Page 625

626 PART THREE DOING FINANCIAL ACCOUNTING

NEL

3. Evaluate each of the following items, stating assumptions or reasons if you wish. Ifthe furniture chain were consolidated with the brewer as of January 1, 2005, whatwould happen on that date to:

NotWould Would No PossibleGo Up Go Down Effect to Tell

a. Consolidated total assetsb. Consolidated shareholders’ equityc. Consolidated net income since

May 1, 2004

4. Looking ahead to the consolidated income for the year ended April 30, 2005, willthe furniture chain’s expected substantial net income contribute significantly to thebrewer’s consolidated net income? Comment on this question.

PROBLEM 9.40 Goodwill and consolidated net income

Foodex Inc. is increasing the vertical integration of its operations by buying out suppli-ers. On July 1, 2005, Foodex purchased 70% of the common shares of Grow FoodFarms, an Alabama supplier of asparagus, celery, and other produce. Grow Food is itselfa bit of a conglomerate because it has extensive landholdings in Atlanta and Dallas, andowns dairies in Wisconsin and Ontario.

On July 1, 2005, Grow Food’s balance sheet showed a net book value of$112,800,000. The aggregate net fair value of individual assets and liabilities was$171,000,000 as at that date. Foodex paid $165,000,000 ($25,000,000 cash, the rest newlyissued shares of Foodex) for its investment in Grow Food.

It is now March 31, 2006, the end of Foodex’s fiscal year. For Foodex, it has been agood year. Grow Food has also done well, reporting a net income of $39,000,000 duringthe nine months ended March 31, 2006, and declaring $15,000,000 in dividends duringthat period. On a nonconsolidated basis, Foodex’s investment in Grow Food stood at$181,800,000 at March 31, 2006, and its net income for the year was $74,200,000.

1. What accounting method is Foodex using for its investment in Grow Foods? Howcan you tell?

2. Two figures that may appear on the consolidated balance sheet of Foodex and GrowFood are “Goodwill arising on consolidation” and “Minority interest liability.” Whyare the fair values of Grow Food’s individual assets relevant for calculating theformer but not the latter?

3. Calculate consolidated net income for the year ended March 31, 2006, as well as youcan with the data provided.

PROBLEM 9.41 Accounting for an acquisition

Winnipeg Merchandisers Ltd. and Red River Stores Ltd. are considering some form ofbusiness combination. Two alternatives are being studied: a purchase of all the votingshares of Red River by Winnipeg or a purchase of 75% of the voting shares of Red Riverby Winnipeg. Management of the two companies wants to know what the combinedcompany’s consolidated balance sheet would look like, based on their present financialinformation. (The final figures will depend on the actual date of the combination.)Here are the two companies’ balance sheets as at a recent date:

hallenging andC

E xtended time

hallenging andC

E xtended time

FinancialAcct-C09-v3 3/28/06 2:56 PM Page 626

CHAPTER 9 LIABILITIES, EQUITY, AND CORPORATE GROUPS 627

NEL

Winnipeg Red River Red RiverBook Values Book Values Fair Values

AssetsCurrent assets $1,124,645 $1,005,789 $1,104,311Noncurrent assets 3,678,872 2,890,003 3,040,722

Liabilities and EquityCurrent liabilities 1,076,554 879,321 899,321Shareholders’ equity 3,726,963 3,016,471

If Winnipeg purchases all of Red River’s voting shares, it is willing to pay $3,400,000,which it will finance by borrowing against the values of its and Red River’s assets. IfWinnipeg purchases only 75% of the voting shares, it is willing to pay only $2,400,000,because the remaining block of 25% would be held by a single person who might inter-fere with Winnipeg’s plans for the combined company.

1. Present the consolidated balance sheet under each of the two combinations beingconsidered:a. 100% purchase of Red River by Winnipeg.b. 75% purchase of Red River by Winnipeg.

2. Write a brief report to the management of the two companies, explaining carefully,and avoiding accounting jargon:a. specifically, what the differences are in the consolidated balance sheets under

the two combinations and why those differences exist; andb. which of the two balance sheets, in your opinion, would be the strongest.

Integrated Problems

Matching Terms

* PROBLEM 9.42 Match liability, equity, and business combinations terms

Match each phrase on the left with the most appropriate phrase on the right.

1. Has no effect on equity at acquisition date a. Income tax-future portion2. Liability arises only on consolidation b. Loan from shareholders3. Left out of the income statement c. Stock dividend4. A name that encompasses debts and d. Purchase method consolidation

some assets e. Equity method5. Financing that isn’t a liability or an equity f. Proprietor’s income tax6. Accrues a share of a noncontrolled g. Minority interest

company’s income h. Financial instruments7. An expense based on analysis of i. Off-balance-sheet

balance sheet accounts j. Indenture8. A promise to meet specified financial

conditions9. A usually informal kind of debt

10. Trying to give something for nothing

FinancialAcct-C09-v3 3/28/06 2:56 PM Page 627

628 PART THREE DOING FINANCIAL ACCOUNTING

NEL

PROBLEM 9.43 Match intercorporate investment terms with their purposes

Match the list of terms in the left column with the purposes in the right column, writtenin deliberately brief and simple terms.

a. Consolidated 1. A buyer and a sellerb. Cost basis 2. Accounted for as onec. Economic entity 3. Don’t count it until we get itd. Equity basis 4. No longer acceptablee. Fair values 5. Parent and subsidiaries togetherf. Goodwill 6. Take credit for influenceg. Minority interest 7. The extra we paidh. Pooling of interests 8. We have a say but not controli. Purchase method 9. What we didn’t buyj. Significant influence 10. Worth of individual parts

Creating and Understanding Financial Statements

PROBLEM 9.44 Outline a talk on consolidated financial statements

You have been asked to give a talk to a group of individual investors on the subject of“What to look for in consolidated financial statements and how to understand what yousee.” Briefly outline the main points in your talk.

PROBLEM 9.45 Effects of new complexities on financial statements

A business commentator remarked: “Double-entry accounting has been around for along time. Maybe too long. In our complex modern world, many events can happen toan enterprise that do not meet the double-entry accounting model. When the CEOborrows money to buy a new home, that is not in the enterprise’s accounting system, norshould it be. So why is anyone surprised that some financing methods don’t fit thedouble-entry model and so aren’t in the balance sheet’s numbers? Accounting transac-tions and adjustments can’t cover everything, and if we tried to make them do that, we’dend up with meaningless balance sheets. So leave the balance sheet alone and just makesure everything is fully disclosed in the notes.”

Respond to the commentator’s remarks.

PROBLEM 9.46 Prepare a full set of statements under GAAP

Macro Ltd. services personal computers. Don Debit, the company’s accountant, hascompiled the following list of balances in the company’s accounts at January 31, 2005,the fiscal year-end.

hallenging andC

E xtended time

FinancialAcct-C09-v3 3/28/06 2:56 PM Page 628

CHAPTER 9 LIABILITIES, EQUITY, AND CORPORATE GROUPS 629

NEL

Macro Ltd.List of Account Balances, January 31, 2005

Accounts payable $ 40,000Accounts receivable 48,000Accumulated depreciation—building 6,000Bank loan 100,000Building 86,000Cash 30,000Depreciation on building for the year 6,000Dividends declared 20,000Dividends payable 5,000General expenses 170,000Investments 16,000Income tax expense 3,000Interest on bank loan for the year 12,000Land 50,000Service fee revenues 213,000Share capital 15,000Shareholders’ loan 60,000Repair supplies used during year 38,000Retained earnings 25,000Gain on sale of plant and equipment 15,000

Here is additional information about some of these figures:

1. Accounts payable will be paid in February 2005 except for $5,000, which will be paidFebruary 1, 2006.

2. Accounts receivable will be collected in February or March 2005.3. The bank loan is payable in yearly payments of $20,000, which are to be made

December 31 of each year for the next five years. Interest on the loan has been paidup to January 31, 2005.

4. A dividend of $20,000 was declared on January 31, 2005, and $15,000 was paid tothe shareholders on the same day. The balance will be paid on December 31, 2005.

5. “Investments” were purchased just before the year-end and consist of the following:

8% term deposits, due in April 2005 $ 4,000Investment in shares of ABC Ltd. 12,000

$16,000

The shares of ABC Ltd. are not publicly traded, so there is no ready market forthem. Macro Ltd. owns 3% of ABC Ltd.’s shares.

6. The shareholders’ loan is due on demand, but no repayment is expected to bemade in the upcoming year. This loan is unsecured.

7. The land and building are recorded at estimated market value, based on anappraisal done on January 2, 2005. The land had cost $40,000; the building had cost$96,000.

8. The bank loan is secured by a first mortgage on the land and building.9. Of the cash, $5,000 is held in trust, as part of a recent sales agreement with a

customer.10. “General expenses” includes $50,000 of a bad-debt expense due to the bankruptcy

of a major customer during the year.11. “Income tax expense” includes $1,000 of tax related to the sale of plant and equip-

ment during the year, future tax of $4,000, and a claim for a tax refund of $2,000.The refund claim is included in accounts receivable.

FinancialAcct-C09-v3 3/28/06 2:56 PM Page 629

12. “Service fee revenues” includes $20,000 of revenue that should have been recordedin the prior year.

13. It turns out that retained earnings includes a future income tax liability of $6,000($2,000 from the beginning of the year and $4,000 from item 11 above). Don didn’tknow where to put it so he just included it in retained earnings.

Prepare a properly classified balance sheet, income statement, and statement ofretained earnings from the above information for the year ended January 31, 2005, aswell as any notes to the financial statements that you feel are necessary to satisfy userneeds and disclosure requirements under GAAP. If it is helpful, assume that a prospec-tive creditor such as a bank will use the financial statements.

630 PART THREE DOING FINANCIAL ACCOUNTING

NEL

FinancialAcct-C09-v3 3/28/06 2:56 PM Page 630

CHAPTER 9 LIABILITIES, EQUITY, AND CORPORATE GROUPS 631

NEL

In mid-2002, Pfizer Inc., the maker of cholesteroldrug Lipitor and virility drug Viagra, acquiredPharmacia Corp., another large drug company,

for U.S.$60 billion. Let’s look at the accounting forsuch a business combination. Pfizer paid forPharmacia with its own newly-issued shares, so inPfizer’s books, the acquisition would be recorded likethis (depending on the actual market values ofPfizer’s shares when issued):

DR Investment in Pharmacia 60 billionCR Share capital 60 billion

There would be no entry in Pharmacia’s books,because all that changed for Pharmacia was itsowners.

In the consolidated financial statements of Pfizer,the accounting would depend on the fair values ofPharmacia’s net assets. At the end of 2001,Pharmacia’s balance sheet showed total assets ofU.S.$22 billion, including U.S.$6 billion for the netassets of Monsanto, which Pharmacia was in theprocess of getting rid of, “spinning it off” to share-holders by issuing a dividend consisting of theMonsanto shares Pharmacia owned. At the end of2001, Pharmacia’s liabilities (without any ofMonsanto’s) were U.S.$10 billion, giving it a netequity value of U.S.$12 billion, or U.S.$6 billion with-out Monsanto. Pharmacia had several importantproducts under development at the time of Pfizer’stakeover, so let’s say that at the middle of 2002,Pharmacia’s net assets had a fair value of about $20 billion without Monsanto, which was almostentirely spun off by then. Let’s say also that theU.S.$4 billion in increased fair value was due only toPharmacia’s assets, that is, that the fair values of itsliabilities equalled their U.S.$10 billion book value.In the consolidated balance sheet of Pfizer at thedate of acquisition, there would thus be:

An increase in consolidated assets other than goodwill of $20 billion

An increase in consolidated goodwill of 50 billion

$70 billion

An increase in consolidated liabilities of 10 billionHigher Pfizer share capital of 60 billion

$70 billion

If the acquisition had been for cash, buyingPharmacia’s shares on the market, Pfizer would havehad to borrow the money (at least until it could issueits own shares on the market to pay off the debt).Pfizer had only about U.S.$1 billion in cash, so itwould have had to borrow the whole U.S.$60 billion.This would be quite impractical, because before theacquisition, Pfizer had assets of about U.S.$40 billionand liabilities of a little more than half that, equity alittle less than half. Such borrowing would havequadrupled its liabilities and raised its debt-equityratio from a little over 1:1 to 4:1. The risks and inter-est costs would have been huge. By issuing shares toPharmacia owners instead, Pfizer transferred the riskto present shareholders, who saw their shareholdingssuddenly diluted by large numbers of Pharmaciashareholders, and to Pharmacia shareholders, whowould have to hope that Pfizer would make their newshares perform well. Pfizer trades on the New YorkStock Exchange: its share prices had been decliningsteadily before the July 15, 2002, announcement ofthe Pharmacia deal, and they continued to declineafter the announcement, picking up a little at theend of the month. (This followed the general patternof share prices in the turbulent mid-2002 stockmarket.)

You can look at Pfizer’s financial statements,MD&A and press releases on www.pfizer.com and seehow the acquisition was accounted for, and how theacquisition of Pharmacia has turned out. Share pricescan be seen on a number of services, or on the NYSEWeb site www.nyse.com. If the numbers are greatlydifferent from those above, and/or if the acquisitionhit any snags (for example, problems with anti-trustcompetition regulators), your discussion of the issuesbelow may change. Both companies’ Web sites hadnews about the acquisition in mid-2002, butPharmacia may no longer have an independent Website by the time you read this.

A July 15, 2002, article about the acquisitioncommented:

The deal comes amid unprecedentedpressure on pharmaceutical companies, asthe industry struggles with a dry spell in itsresearch labs, fights rising competitionfrom generics makers and wrestles with

CASE 9AEVALUATE THE ACCOUNTING FOR A RECENT CORPORATE

ACQUISITION

FinancialAcct-C09-v3 3/28/06 2:56 PM Page 631

632 PART THREE DOING FINANCIAL ACCOUNTING

NEL

intense pressure on prices from govern-ments and private buyers worldwide. As aresult, drug companies are facing slowingor declining revenues and are watchingtheir stocks sink to their lowest level inyears.21

The article speculated that, as the new Pfizer wouldbe the world’s largest drug company measured byrevenue, its rivals (like Bristol-Myers Squibb Co.,Merck & Co. and GlaxoSmithKline PLC) would beunder pressure to find merger partners to keep upwith Pfizer’s growth. On the other hand, the articlealso said that mergers in the pharmaceutical industryhad generally not worked out too well (though prais-ing Pfizer’s earlier acquisition of Warner-Lambert).The stock prices of acquiring companies oftendecline after a merger announcement, indicatingthat stock market traders think promised synergiesand other benefits are unlikely, or that the price paidby the acquiring company was too large. Pfizer (or itsshareholders) paid quite a high price: Pharmacia’s2001 net income was U.S.$1.5 billion, includingU.S.$229 million from Monsanto, being spun off.The goodwill implied in the U.S.$60 billion price wasthus around 40 times Pharmacia’s current earnings.

Under the new rules for accounting for goodwill,Pfizer would write a large portion of theU.S.$50 billion off as a special item if there isevidence that too much was paid. Throughout allthis, and including such a write-off, there would beno effect on Pfizer’s cash flow, indeed it would beincreased by adding Pharmacia’s cash flow.Therefore, even if there were a huge goodwill write-off (and you can check by looking at Pfizer’s 2002 or2003 annual report), there might be little apparentimpact on the company’s performance or shareprice of “paying too much.”

1. Using the above information and anything aboutPfizer available since, discuss the business valueand impact on shareholders of the Pharmaciaacquisition.

2. Discuss the accounting for the acquisition, espe-cially the value attributed to the goodwill andhow the goodwill might be accounted for in lateryears. How might a reader of Pfizer’s financialstatements judge the value and meaning of thegoodwill figure? (If the accounting for the acqui-sition was significantly different from thatoutlined above, identify the differences anddiscuss why they arose.)

3. The amount of goodwill produced in accountingfor such an acquisition could be reduced by(1) assigning more dollars to the fair values ofthe acquired company’s net assets, or (2) reduc-ing the credit to share capital used in recordingthe acquisition, or even (3) just writing the good-will off immediately. Would any of these methodsmake sense?

4. Such an acquisition has either no effect or apositive effect on consolidated cash flow. Thecash flow effects thus might be widely differentfrom the effects on accrual income now or later,and on the debt-equity structure of the balancesheet. Does this divergence indicate weaknessesin the way accrual accounting accounts for business combinations?

5. Many companies struggle with high debt loadscaused by acquiring other companies. Given thePfizer example, why wouldn’t all acquisitions beaccomplished by issuing new shares rather thanpaying in cash?

FinancialAcct-C09-v3 3/28/06 2:56 PM Page 632

CHAPTER 9 LIABILITIES, EQUITY, AND CORPORATE GROUPS 633

NEL

CASE 9BPOOR ACCOUNTING FOR DEBT AND EQUITY?Judge: The witness for the prosecution having

been sworn in, the prosecution maybegin its questions.

Prosecutor: Thank you, Your Honour. Now,Dr. Runciman, would you please stateyour qualifications and experience?

Witness: I have a Ph.D. in accounting and havebeen a professor of accounting at LocalUniversity for 12 years. Much of myresearch and consulting has beenfocused on companies’ accounting forand reporting of their debt and equity.

Prosecutor: Thank you. I hereby file Dr. Runciman’scurriculum vitae with the Court. Now,Dr. Runciman, please state, to beginwith, how a company’s accounting fordebt and equity could potentially harminvestors.

Defence: Objection. Calls for speculation, and ageneral statement may have nothing todo with the facts of this case.

Judge: Sustained. Prosecution will rephrasethe question.

Prosecutor: Dr. Runciman, what are “generallyaccepted accounting principles”?

Witness: These principles, usually referred to asGAAP, are the aggregation of publishedregulations and accepted good prac-tices. They carry official weight in thatthe auditor’s report states whether theauditors believe the financial state-ments have been prepared in accordance with GAAP.

Prosecutor: So companies must follow these GAAP?Witness: No, GAAP are not laws. There may be

more than one acceptable way to dothe accounting, so companies mustdetermine which methods are mostappropriate for their circumstances.Some previous court cases, such asContinental Vending in the U.S., haveheld that following GAAP is less impor-tant than preparing fair financial state-ments, so financial reporting is morecomplicated than just following GAAP.

Prosecutor: Dr. Runciman, what are debt andequity, as far as accounting isconcerned?

Witness: Debt is money owing to parties outsidethe company, resulting from transac-tions or agreements already made.Equity is the owners’ interest in thecompany, as built up over the years byowners’ contributions and earningsretained in the company.

Prosecutor: So if the company has made a promiseto pay an outsider, that promise is adebt.

Witness: Generally, yes. But if the promisedepends on some other event that is yetto happen, the promise is not consid-ered a debt. For example, a promise topay for goods that have not yet beendelivered is not a debt. But it may bedisclosed in the financial statementnotes as a “commitment” or “contin-gent liability.” Also, the company has toevaluate what is likely to be paid tomeet any promise and record that like-lihood. For example, if a promise hasbeen made to pay pensions, thecompany has to estimate how manyemployees will quit, get fired ,or diebefore retirement and so not get apension. The balance sheet reports thebest estimate of what is owed forpensions, not the higher amount thattheoretically has been promised.

Prosecutor: Let’s get to this case. At the March 31date specified in the fraud chargeagainst Scarborough Fair Inc. and itsofficers, the charge this court is consid-ering, were there problems in thecompany’s accounting for its debt andequity in your view?

Witness: There were two problems to start with.One is that the company showed aslong-term debt some debts that it hadto repay within the next year. I estimatethe amount to be $73 million at thespecified balance sheet date. Thesecond is that the company had somepreferred shares that were convertibleto debt, and since the company hadbeen performing relatively poorly, Ithink the preferred shareholders would

FinancialAcct-C09-v3 3/28/06 2:56 PM Page 633

634 PART THREE DOING FINANCIAL ACCOUNTING

NEL

intend to convert the shares, so theyshould have been shown as debt, notequity.

To improve the exposition, some comments by thedefence are reported below even though they wouldbe made later when the defence cross-examined thewitness.

Defence: You said the shares should be shown asdebt because the shareholdersintended to convert them.

Witness: Yes.Defence: Do you have any evidence of that

intent?Witness: There was no evidence at March 31,

but it would have been the rationalthing to do. And since the fraudcharge was laid, some shareholdershave done that.

Defence: So it was rational to convert. Would italso be rational for Scarborough Fairto refinance its long-term debt so thatnothing would have to be paid in thenext year?

Witness: Yes, I suppose so.Defence: You claim that rationality and intent

are appropriate criteria for theaccounting for the preferred shares,but not for the long-term debt. Aren’tyou being inconsistent?

Witness: Well, …Prosecutor: The company and its officers are on

trial here, not the shareholders. Let’sturn to another issue. ScarboroughFair’s balance sheet at March 31reported a $16 million debt to an asso-ciated company, Robinson ProductsLtd. Please comment on that.

Witness: There were two problems. First,Scarborough Fair had guaranteedcertain of Robinson Products’ debts, soa more appropriate debt figure wouldbe the guaranteed debt of $138 million.There should have been a note dis-closure of this contingent liability.Second, Scarborough Fair owned 78%of Robinson Products, so Robinson’saccounts should have been consoli-dated with Scarborough’s, which wouldhave revealed the full extent ofRobinson’s debts, $320 million.

Defence: Are you saying that a potential debtshould have been recorded as anactual one? This contradicts your definition of a debt.

Witness: Scarborough controlled Robinson, soin an economic sense, Robinson’s debtwas Scarborough’s.

Defence: Professor, surely we cannot convictanyone for being a bad economist!(Laughter in the courtroom.) Legally,did Scarborough owe more than$16 million in regard to Robinson?

Prosecutor: Objection. That is a question of law,not accounting.

Judge: Objection upheld. Is the witness sayingthat there was not a problem in thecompany’s debt to Robinson but ratherin the way the corporate group wasreported?

Witness: Yes. People relying on Scarborough’sfinancial statements should have beentold that there was much more to therelationship than $16 million.

Prosecutor: By failing to report the ownership ofRobinson properly, did the defendantsalso report Scarborough’s equityimproperly?

Witness: Yes. Robinson had been sufferinglosses, which Scarborough did notreport because it didn’t consolidateRobinson, so Scarborough’s equityshould have been $38 million lowerdue to its share of those losses.

Defence: Scarborough was using equity account-ing for its relationship with Robinson.Didn’t that mean Scarborough didshow the impact on its equity of theRobinson losses?

Witness: Yes, but …Defence: Thank you.Prosecutor: Dustin Distributing Inc. is another part

of Scarborough Fair’s group. It was89% owned, and was consolidated withScarborough, was it not?

Witness: Yes.Prosecutor: Now, Robinson owed Dustin

$78 million too. What happened to that?Witness: In the consolidated balance sheet, that

$78 million was shown as an accountreceivable. So it looked like an asset,not a debt.

FinancialAcct-C09-v3 3/28/06 2:56 PM Page 634

CHAPTER 9 LIABILITIES, EQUITY, AND CORPORATE GROUPS 635

NEL

Defence: It was not a debt. In fact, if, as you say,Robinson should have been consoli-dated, then both the asset and the debtwould be eliminated anyway as beingwithin the group. Is that right?

Witness: Not exactly. The asset would have beeneliminated, but as the debt was notthere to start with, consolidating wouldnot change that.

Prosecutor: Let’s turn to Scarborough’s directdebts. Was anything missing from itsbalance sheet?

Witness: Yes. First, the company had recognizedrevenue from several large contractsbut had not shown the associatedexpenses because they had not yet beenpaid. Either the revenue was over-reported or the expenses were under-reported. My estimate is that incomeafter tax was over-reported by$29 million because of this under-reporting of debt. Second, thecompany had made promises to issueshares to senior managers under stockoption plans—those promises shouldhave been recorded as compensationexpenses and liabilities. I estimate theunrecorded liability at March 31 to be$110 million, net of some tax effects.

Defence: But the company’s shares were declin-ing in value and since the fraudcharges were laid, the shares have gonedown enormously. Who’d want theshares under option?

Witness: That’s not the point. At March 31, thepromises had been made and shouldhave been recorded. Not only wereliabilities under-stated, but also incomeand retained earnings were over-stated.

Defence: Did the March 31 financial statementnotes disclose the potential additionalshare issues due to the option plan?

Witness: Yes.Defence: And did the diluted earnings per share

calculation take the potential shareissues into account?

Witness: Yes.Defence: So couldn’t anyone reading the finan-

cial statements have made the calcula-tion you just reported?

Witness: Yes, but not easily.Prosecutor: We are about to turn to other witnesses

who will show how the fraud describedin the charges against the defendantswas perpetrated, but I have one morequestion for this witness. WereScarborough Fair’s revenues for theyear ended March 31 too high, in youropinion? If so, what does that have todo with the balance sheet?

Witness: Yes, the revenues were too high.Scarborough Fair had made arrange-ments with a supplier, Canticle Inc., toswap advertising space on their Webpages. Each company would carry adsfor the other company, at no cash costbecause the agreement was that the adshad the same value to both companies.Scarborough Fair recorded the swapagreement by increasing its revenueand its advertising expense at the sametime. In my opinion, the dollar amountwas much too high, because Web adver-tising is not really very valuable.Scarborough Fair recorded a receivablefrom Canticle for Canticle’s advertisingand a payable to Canticle forScarborough’s advertising. From timeto time, the receivable and payablewould be written off against each other.But while the two accounts existed, theimportant working capital ratio wasaffected, and the company couldmanipulate it by choosing when towrite the receivable and payable offagainst each other, and by how much.The debt-equity ratio was also affectedby this accounting.

Defence: How can you criticize this accounting?It made the debts higher than theywould have been, not lower, andincome was not affected. It sounds likegood, conservative accounting.

Discuss the debt and equity accounting issues raisedby this court case.

FinancialAcct-C09-v3 3/28/06 2:56 PM Page 635

636 PART THREE DOING FINANCIAL ACCOUNTING

NEL

NOTES

1. CPR, 2004 annual report, 70–72.

2. E. Church, “Profits seen threatened by rosy pensionnumbers: Off-balance-sheet shortfall could reach state-ments,” The Globe and Mail (May 7, 2002), B9.

3. Various contributors, “Enron: Running on empty,”Business Week (December 10, 2001), 80–82.

4. C. Byrd, I. Chen, and J. Smith, Financial Reporting inCanada 2005 (Toronto: Canadian Institute ofChartered Accountants, 2005), 365–366.

5. “Note W,” IBM, 2004 Annual Report. Online,www.ibm.com.

6. Note 16, GM 2004 annual report. Online,www.gm.com.

7. CPR, 2004 annual report, 72.

8. Byrd, Chen and Smith, 365.

9. Berkshire Hathaway 2001 Annual Report, (PDFversion), 1.

10. Ibid., 2

11. “Forbes 400 Richest Americans in 2001,” availableonline at www.forbes.com.

12. Byrd, Chen and Smith, 489.

13. F. Taylor (Vox), “Expensing options is a moveforward,” The Globe and Mail (May 17, 2002), B9.

14. M. Ingram, “Any way you look at it, options are anexpense,” The Globe and Mail (May 2, 2002), B7.

15. “Badly in need of repair,” The Economist (May 4, 2002),66–68.

16. N. Byrnes, “Five Ways to Avoid More Enrons,” BusinessWeek (February 18, 2002), 36–37.

17. Byrd, Chen, and Smith, 345.

18. CPR, 2004 annual report, 63–66.

19. CICA Accounting Handbook, Section 3855.12.

20. C. Byrd, I. Chen, and J. Smith, Financial Reporting inCanada 2005 (Toronto: Canadian Institute ofChartered Accountants, 2005), 223.

21. R. Frank and S. Hensley, “Pfizer to buy Pharmacia for$60 billion in stock,” The Wall Street Journal, reprintedin The Globe and Mail (July 15, 2002), B5.

FinancialAcct-C09-v3 3/28/06 2:56 PM Page 636